Всероссийская олимпиада школьников муниципальный этап по физике 10 класс: Всероссийская олимпиада по физике, задания

Содержание

Школьный этап Всероссийской олимпиады школьников: Порядок просмотра результатов

Требования к проведению школьного этапа


Всероссийской олимпиады школьников 2020/2021 учебного года в онлайн-формате

 

1. Школьный этап всероссийской олимпиады школьников в онлайн-формате (далее – олимпиада) проводится по 6 общеобразовательным предметам (физика, химия, биология, математика, информатика, астрономия) с использованием дистанционных информационно-коммуникационных технологий в части организации выполнения олимпиадных заданий, организации проверки и оценивания выполненных олимпиадных работ, анализа олимпиадных заданий и их решений, показа выполненных олимпиадных работ, при подаче и рассмотрении апелляций. Участники выполняют олимпиадные задания в тестирующей системе.

2. Регионы проведения олимпиады: Тюменская область, Нижегородская область, Оренбургская область, Самарская область, Воронежская область, Республика Дагестан, г. Севастополь, г. Сочи.

3. Доступ к заданиям по данному предмету предоставляется участникам:

7-11 классы – в течение одного дня, указанного в графике школьного этапа олимпиады, в период с 8:00 до 20:00 по московскому времени;
4-6 классы – в течение трёх календарных дней, начиная с дня, указанного в графике школьного этапа олимпиады, в период с 8:00 первого дня до 20:00 третьего дня (по московскому времени).

4. Участники школьного этапа олимпиады вправе выполнять олимпиадные задания, разработанные для более старших классов по отношению к тем, в которых они проходят обучение.

5. Вход участника в тестирующую систему осуществляется по индивидуальному коду (для каждого предмета отдельный код), который направляется (дистанционно выдается) каждому участнику в его образовательной организации. Этот индивидуальный код предоставляет участнику также доступ к его результатам после завершения олимпиады. Инструкция о порядке доступа в тестирующую систему публикуется на официальном сайте Образовательного центра «Сириус» sochisirius.ru.

6. Время, отведенное на выполнение заданий для каждого общеобразовательного предмета и класса, указывается непосредственно в тексте заданий, а также публикуется на официальном сайте Образовательного центра «Сириус» и странице олимпиады в регионе. Участник олимпиады может приступить к выполнению заданий в любое время, начиная с 8:00. Выполненная работа должна быть сдана участником до окончания отведенного времени на выполнение, но не позже 20:00. В случае, если работа не была сдана участником до окончания отведенного времени на выполнение, несданная работа будет автоматически принята в систему и направлена на проверку. Все не сданные в 20:00 последнего дня работы будут автоматически приняты в систему и направлены на проверку.

7. Требования к порядку выполнения заданий школьного этапа олимпиады по данному предмету и классу публикуются на официальном сайте Образовательного центра «Сириус» не позднее, чем за 5 календарных дней до даты проведения олимпиады. Требования определяют: время, отведенное на выполнение заданий, комплекты заданий по классам (параллелям), наличие или отсутствие аудио- и видеофайлов.

8. Участники выполняют олимпиадные задания индивидуально и самостоятельно. Запрещается коллективное выполнение олимпиадных заданий, использование посторонней помощи (родители, учителя, сеть Интернет).

9. Образовательные организации получают доступ к индивидуальным кодам участников не позднее 5 календарных дней до даты проведения олимпиады в соответствии с инструкцией на официальном сайте Образовательного центра «Сириус».

10. Участники олимпиады получают доступ к своим результатам не позднее 10 календарных дней после даты проведения олимпиады в соответствии с инструкцией на официальном сайте Образовательного центра «Сириус».

11. Вопросы участников олимпиады по техническим ошибкам, связанным с оценкой олимпиадной работы или подсчетом баллов, принимаются в течение двух календарных дней после публикации результатов олимпиады по соответствующему общеобразовательному предмету и классу по процедуре, описанной на официальном сайте Образовательного центра «Сириус». Вопросы участников будут рассмотрены и технические ошибки будут устранены в случае их подтверждения не позднее семи календарных дней после поступления.

12. Итоговые результаты школьного этапа олимпиады по каждому общеобразовательному предмету подводятся независимо для каждого класса и направляются в регион (региональному координатору).

Всероссийская олимпиада по физике. Муниципальный этап.

Муниципального этап Всероссийской олимпиады школьников

по физике в Коломенском Политехе

В воскресенье, 16 декабря 2018 года, Коломенский институт (филиал) Московского политехнического университета принимал участников муниципального этапа Всероссийской олимпиады школьников по физике.

В олимпиаде приняли участие около 40 школьников из 10-11 классов школ Коломенского городского округа: учащиеся гимназии № 2 «Квантор», лицея № 4, гимназий № 8 и № 9, средних общеобразовательных школ №№ 7, 10, 12, 14, 15, 18, 20, 21 и 24, а также Карасевской, Песковской, Радужненской, Сергиевской средних общеобразовательных школ.

Состав комиссии по проведению олимпиады на площадке Коломенского института (филиала) Московского политеха:

Председатель комиссии — Быкова Ю. Н., методист МБУ ДПО «Учебно-методический центр «Коломна», учитель физики МБОУ «Гимназия № 2 «Квантор».

Члены комиссии:

1. Андриянова С. В., МБОУ СОШ № 7

2. Молчанова И. Г., МБОУ «Гимназия № 8»

3. Черникова Л. В., МБОУ СОШ № 24

4. Устинова О. В., МБОУ СОШ № 28

5. Гурова А. А., ОСШ № 1

6. Мавлютова О. В, МОУ Карасёвская СОШ

Ответственные лица по организации и проведению олимпиады на площадке Коломенского института (филиала) Политеха: зав. кафедрой ЕНД Фирсова Е. В.; доцент Леонова А. В.; старший преподаватель Виденина Наталья Владимировна; старший преподаватель Кузнецов Владимир Аркадьевич; старший преподаватель Лобанов Александр Александрович

Всероссийская олимпиада школьников — массовое ежегодное мероприятие по работе с одаренными школьниками в системе российского образования. Олимпиада проводится для обучающихся государственных, муниципальных и негосударственных образовательных организаций, которые реализуют образовательные программы основного общего и среднего общего образования.

Олимпиада проводится в течение учебного года с сентября по май в установленные сроки и включает четыре этапа: школьный, муниципальный, региональный и заключительный. Заключительный этап проводится в субъектах Российской Федерации, отобранных на основании заявок.

Победители и призеры заключительного этапа получают диплом, дающий право поступления при наличии аттестата без экзаменов в любой университет Российской Федерации по профилю олимпиады и награждаются специальной премией Правительства Российской Федерации.

Организатором олимпиады является Минобрнауки России, утверждающий состав Центрального оргкомитета и составы Центральных предметно-методических комиссий.

ЖЕЛАЕМ ШКОЛЬНИКАМ ДАЛЬНЕЙШИХ ПОБЕД!

    

 

 

ГБОУ Московской области «Сергиево–Посадский физико–математический лицей» » Русаков Анатолий Васильевич

Название мероприятия

Призёры (фамилия, имя, класс, результат)

год

1

Районная олимпиада

Марковцев Вадим , 11 класс, 2 место

2006

2

Районная олимпиада

Рябченко Игорь, 11 класс, 3 место

2006

3

Областная олимпиада

Алексеенко Андрей, 10 кл., диплом 2 степени

2006

4

Областная олимпиада

Марковцев Вадим, 11 класс,  диплом 2 степени

2006

5

Региональная олимпиада

Марковцев Вадим, 11 класс,  диплом 1 степени

2006

6

Всероссийская олимпиада (федеральный этап)

Марковцев Вадим, 11 класс,  диплом 2 степени

2006

7

Всероссийская олимпиада (заключительный этап)

Марковцев Вадим, 11 класс,  диплом 3 степени

2006

8

Интернациональная олимпиада «Туймаада»

Марковцев Вадим, 11 класс,  диплом 2 степени

2006

9

Районная олимпиада

Алексеенко Андрей, 11 класс, 2 место

2007

10

Областная олимпиада

Алексеенко Андрей, 11 кл.,  диплом 2 степени

2007

11

Областная олимпиада

Булычева Ксения, 11 класс,  диплом 1 степени

2007

12

Областная олимпиада

Стрельцов Григорий, 11 кл.,  диплом 2 степени

2007

13

Региональная олимпиада

Хегай Олег, 11 класс, диплом 3 степени

2007

14

Районная олимпиада

Вдовина Евгения , 9 класс, 3 место

2008

15

Муниципальный этап Всероссийской олимпиады школьников по физике

Вдовина Евгения , 11 класс, 1 место

2010

16

Региональный этап Всероссийской олимпиады школьников по физике

Вдовина Евгения , 11 класс,  диплом призера

2010

17

Московская городская олимпиада по физике

Вдовина Евгения, 11 класс, диплом 3 степени

2010

18

Муниципальный этап Всероссийской олимпиады школьников по физике

Склонин Илья, 9 класс,  диплом победителя

2011

19

Муниципальный этап Всероссийской олимпиады школьников по физике

Исмайлова Шараф, 9 класс,  диплом призера

2011

20

Муниципальный этап Всероссийской олимпиады школьников по физике

Дрожжилов Петр, 9 класс,  диплом призера

2011

21

Муниципальный этап Всероссийской олимпиады школьников по физике

Дианова Анастасия, 9 класс,  диплом призера

2011

22

Турнир имени М.В.Ломоносова

Тормагов Тимофей, 9 класс,  диплом призера

2011

23

Турнир имени М.В.Ломоносова

Мазаев Вадим, 9 класс,  диплом призера

2011

24

Турнир имени М.В.Ломоносова

Константинов Федор, 9 класс,  диплом призера

2011

25

Турнир имени М.В.Ломоносова

Дианова Анастасия, 9 класс,  диплом призера

2011

26

Региональный этап Всероссийской олимпиады школьников по физике

Склонин Илья, 9 класс,  диплом призера

2011

27

Региональный этап Всероссийской олимпиады школьников по физике

Дианова Анастасия, 9 класс,  диплом призера

2011

28

Региональный этап Всероссийской олимпиады школьников по физике

Дрожжилов Петр, 9 класс,  диплом призера

2011

29

Международная олимпиада «Туймаада»

Склонин Илья, 9 класс,  бронзовая медаль

2011

30

Международная олимпиада «Туймаада»

Дианова Анастасия, 9 класс,  бронзовая медаль

2011

31

Муниципальный этап Всероссийской олимпиады школьников по физике

Тормагов Тимофей, 10 класс,  диплом призера

2012

32

Муниципальный этап Всероссийской олимпиады школьников по физике

Константинов Федор, 10 кл., диплом победителя

2012

33

Муниципальный этап Всероссийской олимпиады школьников по физике

Склонин Илья, 10 класс,  диплом призера

2012

34

Муниципальный этап Всероссийской олимпиады школьников по физике

Дианова Анастасия, 10 класс,  диплом призера

2012

35

Муниципальный этап Всероссийской олимпиады школьников по физике

Зотов Михаил, 10 класс,  диплом призера

2012

36

Муниципальный этап Всероссийской олимпиады школьников по физике

Карулин Евгений, 10 класс,  диплом призера

2012

37

Региональный этап Всероссийской олимпиады школьников по физике

Константинов Фёдор, 10 класс,  диплом призера

2012

38

Региональный этап Всероссийской олимпиады школьников по физике

Дианова Анастасия, 10 класс,  диплом призера

2012

39

Турнир имени М.В.Ломоносова

Константинов Федор, 10 класс,  диплом призера

2012

40

Турнир имени М.В.Ломоносова

Склонин Илья, 10 класс,  диплом призера

2012

41

Турнир имени М.В.Ломоносова

Жуков Ярослав, 10 класс,  диплом призера

2012

42

Турнир имени М.В.Ломоносова

Дианова Анастасия, 10 класс,  диплом призера

2012

43

Международная олимпиада «Туймаада»

Константинов Федор, 10 кл., бронзовая медаль

2012

44

Муниципальный этап Всероссийской олимпиады школьников по физике

Склонин Илья, 11 класс,  диплом победителя

2013

45

Муниципальный этап Всероссийской олимпиады школьников по физике

Тормагов Тимофей,  11 класс,  диплом призера

2013

46

Муниципальный этап Всероссийской олимпиады школьников по физике

Мазаев Вадим,  11 класс,  диплом призера

2013

47

Муниципальный этап Всероссийской олимпиады школьников по физике

Смелов Юрий,  11 класс,  диплом призера

2013

48

Муниципальный этап Всероссийской олимпиады школьников по физике

Дианова Анастасия ,  11 класс,  диплом призера

2013

49

Муниципальный этап Всероссийской олимпиады школьников по физике

Константинов Федор, 11 класс,  диплом призера

2013

50

Муниципальный этап Всероссийской олимпиады школьников по физике

Тормагов Тимофей,  11 класс,  диплом призера

2013

51

Международная Жаутыковская олимпиада

Константинов Федор, 11 кл., бронзовая медаль

2013

52

Международная Жаутыковская олимпиада

Склонин Илья, 11 класс, бронзовая медаль

2013

53

Региональный этап Всероссийской олимпиады школьников по физике

Константинов Фёдор, 11 класс,  диплом призера

2013

54

Муниципальный этап Всероссийской олимпиады школьников по физике

Крохалев Олег,  9 класс,  диплом призера

2014

55

Муниципальный этап Всероссийской олимпиады школьников по физике

Жучкова Наталья ,  9 класс,  диплом призера

2014

56

Муниципальный этап Всероссийской олимпиады школьников по физике

Никишова Анастасия,  9 класс,  диплом призера

2014

57

Муниципальный этап Всероссийской олимпиады школьников по физике

Решетников Андрей,  9 класс,  диплом призера

2014

58

Муниципальный этап Всероссийской олимпиады школьников по физике

Горьков Анатолий,  9 класс,  диплом победителя

2014

59

Региональный этап Всероссийской олимпиады школьников по физике

Крохалев Олег,  9 класс,  диплом призера

2014

60

Муниципальный этап Всероссийской олимпиады школьников по физике

Горьков Анатолий,  10 кл.,  диплом победителя

2015

61

Муниципальный этап Всероссийской олимпиады школьников по физике

Шишкин Сергей,  10 класс,  диплом призера

2015

62

Муниципальный этап Всероссийской олимпиады школьников по физике

Крохалев Олег,  10 класс,  диплом призера

2015

63

Региональный этап Всероссийской олимпиады школьников по физике

Горьков Анатолий,  10 класс,  диплом призера

2015

64

Вторая международная олимпиада по экспериментальной физике

Горьков Анатолий,  10 класс, диплом 3 степени и бронзовая медаль

2015

65

Региональный этап Всероссийской олимпиады школьников по физике

Крохалев Олег, 10 класс,  похвальная грамота

2015

66

 «Турнир имени М.В.Ломоносова – 2015»

Алексеева Маргарита, 10 класс, грамота

2015

67

 «Турнир имени М.В.Ломоносова – 2015»

Горьков Анатолий,  10 класс,  грамота

2015

68

Международная олимпиада  по экспериментальной физике

Горьков Анатолий, 11 класс, диплом участника 

2016

69

Международная олимпиада  по экспериментальной физике

Крохалев Олег, 11 класс, диплом участника 

2016

70

Муниципальный этап Всероссийской олимпиады школьников по физике

Горьков Анатолий, 11 класс,  диплом призера

2016

71

Муниципальный этап Всероссийской олимпиады школьников по физике

Решетников Андрей, 11 класс,  диплом призера

2016

72

 «Турнир имени М.В.Ломоносова – 2016»

Горьков Анатолий, 11 класс, грамота

2016

73

 «Турнир имени М.В.Ломоносова – 2016»

Крохалев Олег, 11 класс, грамота

2016

74

 «Турнир имени М.В.Ломоносова – 2016»

Панин Иван, 11 класс, грамота

2016

75

Международная Жаутыковская олимпиада

Горьков Анатолий, 11 класс, диплом участника 

2016

76

Международная Жаутыковская олимпиада

Крохалев Олег, 11 класс, диплом участника 

2016

77

Региональный этап Всероссийской олимпиады школьников по физике

Горьков Анатолий,  11 класс,  диплом призера

2016

78

Международная олимпиада  по экспериментальной физике

Рева Максим, 9 класс, диплом 3 степени и бронзовая медаль 

2017

79

Муниципальный этап Всероссийской олимпиады школьников по физике

Рева Максим,  9 класс,  диплом победителя

2017

80

Муниципальный этап Всероссийской олимпиады школьников по физике

Арустамов Павел , 9 класс,  диплом призера

2017

81

Муниципальный этап Всероссийской олимпиады школьников по физике

Насаченко Максим,  9 класс,  диплом призера

2017

82

Региональный этап Всероссийской олимпиады школьников по физике

Рева Максим,  9 класс,  диплом призера

2017

83

Заключительный этап Всероссийской олимпиады школьников по физике

Рева Максим,  9 класс,  диплом призера

2017

84

 «Турнир имени М.В.Ломоносова – 2017»

Рева Максим, 10 класс, грамота

2018

85

Муниципальный этап Всероссийской олимпиады школьников по физике

Рева  Максим, 10 класс, диплом победителя

2018

86

Муниципальный этап Всероссийской олимпиады школьников по физике

Козлов Максим, 10 класс, диплом призера

2018

87

Региональный этап Всероссийской олимпиады школьников по физике

Рева  Максим, 10 класс, диплом победителя

2018

88

Заключительный этап Всероссийской олимпиады школьников по физике

Рева Максим,  10 класс,  диплом участника

2018

92

Муниципальный этап Всероссийской олимпиады школьников

Рева Максим,  11 класс,  диплом победителя

2019

93

Муниципальный этап Всероссийской олимпиады школьников

Козлов Максим,  11 класс,  диплом призера

2019

94

Муниципальный этап Всероссийской олимпиады школьников

Буланов Михаил,  11 класс,  диплом призера

2019

95

Муниципальный этап Всероссийской олимпиады школьников

Калиничев Игорь,  11 класс,  диплом призера

2019

96

Региональный этап Всероссийской олимпиады школьников

Рева Максим,  11 класс,  диплом призера

2019

97

Муниципальный этап Всероссийской  олимпиады школьников по астрономии

Севастьянов Костя, 9 класс, победитель

2020

98

Муниципальный этап Всероссийской олимпиады школьников

Ильина Александра, 9 класс, призер

2020

99

Муниципальный этап Всероссийской олимпиады школьников

Кузьменков Никита, 9 класс, призер

2020

100

Муниципальный этап Всероссийской олимпиады школьников

Кондратов Денис, 9 класс, призер

2020

101

Муниципальный этап Всероссийской олимпиады школьников

Севастьянов Костя, 9 класс, победитель

2020

102

Региональный этап Всероссийской олимпиады школьников по астрономии

Севастьянов Костя, 9 класс, диплом победителя

2020

103

Региональный этап Всероссийской олимпиады школьников по физике

Севастьянов Костя, 9 класс, диплом призер

2020

Региональный этап | Этапы всероссийской олимпиады школьников

Приказ МО Саратовской области  «Об утверждении результатов регионального этапа всероссийской олимпиады школьников по каждому общеобразовательному предмету на территории Саратовской области в 2019/2020 учебном году»

Иностранные языки: Итоговые протоколы: испанский, итальянский, китайский  Работы участников

Немецкий язык: Итоговые протколы, Работы участников: письменные, устные

Технология: Итоговые протоколы, Работы участников: 9 класс, 10 класс, 11 класс

Экономика: Итоговые протоколы, Работы участников: 9 класс, 10-11 классы

Английский язык: Итоговый протокол, Работы участников: 9 класс, 10 класс, 11 класс

Основы безопасности жизнедеятельности: Итоговые протоколы

Физическая культура: Итоговые протоколы, Работы участников: юноши, девушки

История: Итоговые протколы, Работы участников

Математика: Итоговые протоколы, Работы участников: 8 класс, 9 класс, 10 класс, 11 класс

Искусство: Итоговые протоколы, Работы участников: 9 класс, 10 класс, 11 класс

Химия: Итоговые протоколы, Работы участников: 9 класс, 10 класс, 11 класс

Литература: Итоговые протоколы, Работы участников: 9 класс, 10 класс, 11 класс

Биология: Итоговые протоколы, Работы участников: 9 класс, 10 класс, 11 класс

География: Итоговые протоколы, Работы участников: 9 класс, 10 класс, 11 класс

Физика: Итоговые протоколы, Работы участников: 7 класс, 8 класс, 9 класс, 10 класс, 11 класс

Экология:  Итоговые протоколы, Работы участников:  9 класс, 10 класс, 11 класс

Астрономия: Итоговые протоколы, Работы участников: 9 класс, 10 класс, 11 класс

Право: Итоговые протоколы, Работы участников: 9 класс, 10 класс, 11 класс

Информатика: Итоговые протоколы

Обществознание: Итоговые протоколы, Работы участников: 9 класс, 10 класс, 11 класс

Русский язык: Итоговые протоколы, Работы участников

Французский язык: Итоговый протокол, Работы участников

 

Подведены итоги школьного этапа всероссийской олимпиады школьников по физике

В школьном этапе всероссийской олимпиады школьников по физике в 2018 — 2019 учебном году приняли участие 2 115 учащихся 7-11 классов (в 2017-2018 учебном году – 2 111 чел.) из 50 общеобразовательных учреждений города (№№ 1, 2, 3, 4, 5, 6, 7, 8, 9, 10, 11, 12, 13, 14, 15, 16, 17, 18, 19, 21, 22, 23, 24, 25, 26, 28, 29, 31, 32, 33, 35, 36, 38, 39, 40, 41, 43, 44, 45, 46, 47, 48, 49, 50, 56, КМЛ, ШИЛИ, АПКМК, «Ганзейская ладья», «Альбертина»), а также 13 пятиклассников (ОУ №№ 19, 28, 40) и 15 шестиклассников (ОУ №№ 1, 9, 18, 40, 50), выполнявших работы за курс 7 класса.

 

Победителями школьного этапа олимпиады стали 126 учащихся, призерами – 343 учащихся.

Протоколы проведения олимпиады в общеобразовательных учреждениях и скан-копии работ победителей и призеров участников олимпиады по физике должны быть размещены на официальных сайтах общеобразовательных учреждений в срок до 4 октября 2018 года (включительно).

В соответствии с Порядком проведения всероссийской олимпиады школьников в муниципальном этапе олимпиады по каждому общеобразовательному предмету принимают индивидуальное участие:

– учащиеся 7-11 классов – участники школьного этапа олимпиады текущего учебного года, набравшие необходимое для участия в муниципальном этапе олимпиады количество баллов, установленное организатором муниципального этапа олимпиады;

– победители и призёры муниципального этапа олимпиады предыдущего учебного года, продолжающие обучение в организациях, осуществляющих образовательную деятельность по образовательным программам основного общего и среднего общего образования.

Количество баллов, необходимых для участия в муниципальном этапе олимпиады по физике в 2018-2019 учебном году:

7 класс – 31

8 класс – 22

9 класс – 22

10 класс – 22

11 класс – 22

В число участников муниципального этапа по каждому предмету по решению оргкомитета могут быть включены по одному участнику от общеобразовательного учреждения, обучающиеся которого не набрали необходимое количество баллов для прохождения на муниципальный этап по данному предмету, при условии, что они набрали не менее 40% от максимально возможного количества баллов.

К муниципальному этапу олимпиады допущены 286 учащихся 7-11 классов, в том числе 53 победителя и призера муниципального этапа 2017-2018 учебного года, а также 1 учащийся 5 класса.

Поздравляем учащихся, набравших необходимое количество баллов, и желаем им успеха на муниципальном этапе!

СПРАВОЧНО:

Муниципальный этап олимпиады по физике состоится 07 декабря 2018 года. Пункты проведения олимпиады будут сообщены дополнительно.

Всероссийская олимпиада школьников и московская олимпиада школьников

Уважаемые посетители, с результатами предыдущих периодов проведения ВсОШ можно ознакомится в разделе Результативность обучения — Олимпиады и конкурсы.

Долгожданная победа на заключительном этапе олимпиады.

подробнее

обновлено: 27.05.2021

Составлено расписание пригласительного (пробного) этапа всероссийской олимпиады школьников 2021/22 учебного года в Москве. Соревнование пройдет в дистанционной форме с 13 апреля по 25 мая 2021 года. Для участия в олимпиаде по предмету в каждом классе дается один день с 10.00 до 21.00. На олимпиаду приглашаются ученики столичных школ из 3-10 классов. Большинство предметов проводится начиная с 4 класса, в русском и математике также могут принимать участие третьеклассники. Задания по искусству (МХК), физике и экономике подготовлены для учеников 6-10 классов, по химии – 7-10 классов.

подробнее

обновлено: 06.04.2021

Подведены итоги регионального этапа всероссийской олимпиады школьников. Победителем стал ученик 10 В класса П. Вениамин. Ему удалось стать лучшим в Москве по предмету ИТАЛЬЯНСКИЙ ЯЗЫК.

подробнее

обновлено: 16.03.2021

Инструкция по процедуре регистрации.

подробнее

обновлено: 11.02.2021

Дорогие друзья! Мы рады предоставить Вашему вниманию список учеников школы 1811, показавших отличные результаты! И от души хотим поблагодарить педагогов, подготовивших ребят. Спасибо учителям за терпение и настойчивость, за поддержку и веру в успех! Мы не сомневаемся, что и в региональном этапе ваши ученики продемонстрируют твердые знания. подробнее

обновлено: 25.01.2021

Уважаемые участники, обращаем ваше внимание, что форма проведения олимпиады очная и не предусматривает резервных дней.

подробнее

обновлено: 20.01.2021

25 старшеклассников школы №1811 приглашены на региональный этап всероссийской олимпиады школьников по русскому языку, литературе, ОБЖ, английскому языку, обществознанию, праву, искусству, итальянскому языку, математике, робототехнике, экологии и физике. В этом году тур пройдет очно на территории своей школы с видеофиксацией.

подробнее

обновлено: 14.01.2021

Московская олимпиада школьников для учеников 5–11-х классов приглашает участников! Отборочный этап пройдет в онлайн-формате. Для участия нужно зарегистрироваться. Соревнование пройдет по 22 направлениям в онлайн-формате.

подробнее

обновлено: 04.12.2020

Олимпиада продолжается до 15 декабря. Участие в формате видеоконференций, проводимых сотрудниками Школы№1811 Ссылки на конференции высылает классный руководитель. На все ваши вопросы ответят ответственные в корпусах. Ссылку на сайт Олимпиады и подробное расписание представляем вашему вниманию:

подробнее

обновлено: 04.12.2020

В этом году олимпиада в Москве пройдет в режиме онлайн. Необходимо заранее ознакомиться с особенностями участия на странице олимпиады:

подробнее

обновлено: 08.11.2020

На заседании городского оргкомитета 9 октября 2020 года подвели итоги школьного этапа всероссийской олимпиады в Москве по английскому языку, биологии, основам безопасности жизнедеятельности (ОБЖ), русскому языку, физике, физической культуре, французскому языку и экологии. Опубликованы граничные баллы, которые необходимо было набрать на турах, проходивших с 10 по 30 сентября, для получения статусов победителя и призера. Для получения подробной информации пройдите по ссылке:

подробнее

обновлено: 17.10.2020

Пятеро учащихся старших классов школы 1811 стали победителями!

подробнее

обновлено: 21.01.2020

До конца октября в Москве проходит школьный этап олимпиады школьников. для тех, кто по каким-либо причинам не сможет поучаствовать в очной форме в своей школе, проводится школьный интернет-этап http://vos.olimpiada.ru/2017/school/online

подробнее

обновлено: 18.09.2017

Всероссийская олимпиада школьников в 2016/17 учебном году проводится по 24 общеобразовательным предметам в 4 этапа: школьный, муниципальный, региональный и заключительный.

подробнее

обновлено: 30.11.2016

Олимпиады — МАОУ «Лицей №6»

 

Информация, нормативная правовая база:

Информационный портал ВОШ

Раздел ВОШ на сайте комитета образования администрации города Тамбова

Раздел ВОШ на сайте управления образования и науки Тамбовской области

 

Сроки и места проведения олимпиад 2020/2021: 

Школьный этап

Муниципальный этап

Региональный этап

 

Результаты ВОШ — 2020/2021 учебный год

Результаты регионального этапа

Поздравляем призеров и победителей Регионального этапа ВсОШ 2020-2021

Математика 

Астапова Анастасия (9А класс) — Призер, Учитель — Желтова О.Н. 

Гарницкий Александр (9А класс) — Призер, Учитель — Желтова О.Н. 

Бергман Валерий (10А класс) — Призер, Учитель — Немченко М.Г.

Болдырева Ольга (10А класс) — Призер, Учитель — Немченко М.Г.

Добрынина Екатерина (10А класс) — Призер, Учитель — Немченко М.Г.

Иванникова Анастасия (10А класс) — Призер, Учитель — Немченко М.Г.

Котлов Артём (10А класс) — Призер, Учитель — Немченко М.Г.

Никифорова Анна (11А класс) — Призер, Учитель — Немченко М.Г.

История 

Гарницкий Александр (9А класс) — Призер, Учитель — Махрачев Г.С.

Шутилин Дмитрий (11Б класс) — Призер, Учитель — Обухова Л.И.

Право 

Сурков Дмитрий (10Б класс) — Призер, Учитель — Обухова Л.И.

Борзыкин Артем (11Б класс) — Призер, Учитель — Обухова Л.И.

Маркин Дмитрий (11Б класс) — Призер, Учитель — Обухова Л.И.

Обществознание 

Гарницкий Александр (9А класс) — Победитель, Учитель — Махрачев Г.С.

Бергман Валерий (10А класс) — Призер, Учитель — Махрачев Г.С.

Ефименко Сергей (10Б класс) — Призер, Учитель — Обухова Л.И.

Сорокин Дмитрий (11Б класс) — Призер, Учитель — Обухова Л.И.

География 

Ефименко Сергей (10Б класс) — Призер, Учитель — Васильева Н.Е.

Забелина Софья (10Б класс) — Призер, Учитель — Васильева Н.Е.

Экономика 

Моисейкин Владимир (9Б класс) — Призер, Учитель — Алексашина Е.В.

Сурков Дмитрий (10Б класс) — Призер, Учитель — Алексашина Е.В.

Матвеев Леонид (10А класс) — Призер, Учитель — Алексашина Е.В.

Архангельская Полина (11Б класс) — Призер, Учитель — Алексашина Е.В.

Биология

Журавлева Дария (10В класс) — Призер, Учитель — Белов Р.Н.

Гладышева Варвара (11А класс) — Призер, Учитель — Белов Р.Н. 

Макаров Даниил (11А класс) — Призер, Учитель — Белов Р.Н.

Основы безопасности жизнедеятельности

Казанцева Софья  (9В класс) — Призер, Учитель — Горелкин Ю.В.

Левочкин Андрей  (8А класс) — Победитель, Учитель — Горелкин Ю.В.

Хомутников Дмитрий (8Б класс) — Победитель, Учитель — Горелкин Ю.В.

Котлов Артём (10А класс) — Победитель, Учитель — Горелкин Ю.В.

Андронова Полина (10А класс) — Победитель, Учитель — Горелкин Ю.В.

Вальдман Игорь (10А класс) — Победитель, Учитель — Горелкин Ю.В.

Сивохина Анастасия (10А класс) — Победитель, Учитель — Горелкин Ю.В.

Винокурова Ксения а (11А класс) — Победитель, Учитель — Горелкин Ю.В.

Макаров Даниил (11А класс) — Победитель, Учитель — Горелкин Ю.В.

Стародубова Алёна (11А класс) — Победитель, Учитель — Горелкин Ю.В.

Экология 

Попкова Екатерина (10В класс) — Призер, Учитель — Белов Р.Н.

Ветрова Ольга (11А класс) — Призер, Учитель — Белов Р.Н.

Гладышева Варвара (11А класс) — Призер, Учитель — Белов Р.Н.

Котова Владислава (11А класс) — Призер, Учитель — Белов Р.Н.

Макаров Даниил (11А класс) — Победитель, Учитель — Белов Р.Н.

Физика

Белоусов Николаевич (9А класс) — Призер, Учитель — Якунин В.И.

Богословская Анна (9А класс) — Призер, Учитель — Якунин В.И.

Выгузов Андрей (9А класс) — Призер, Учитель — Якунин В.И. 

Козакова Анна (9А класс) — Призер, Учитель — Якунин В.И.

Яковлева Наталья (9А класс) — Призер, Учитель — Якунин В.И.

Бергман Валерий (10А класс) — Призер, Учитель — Меньших С.А.

Болдырева Ольга (10А класс) — Призер, Учитель — Меньших С.А.

Волков Алексей (10А класс) — Призер, Учитель — Меньших С.А.

Иванникова Анастасия (10А класс) — Призер, Учитель — Меньших С.А.

Котлов Артём (10А класс) — Призер, Учитель — Меньших С.А.

Шабанов Дмитрий (10А класс) — Призер, Учитель — Меньших С.А.

Шульгин Константин (10А класс) — Призер, Учитель — Меньших С.А.

Никифорова Анна (11А класс) — Призер, Учитель — Якунин В.И.

Астрономия

Гарницкий Александр (9А класс) — Призер, Учитель — Меньших С.А.

Котлов Артём (10А класс) — Призер, Учитель — Меньших С.А.

Рогова Александра (10А класс) — Призер, Учитель — Меньших С.А.

Сивохина Анастасия (10А класс) — Призер, Учитель — Меньших С.А.

Бергман Валерий (10А класс) — Призер, Учитель — Меньших С.А.

Никифорова Анна (11А класс) — Призер, Учитель — Меньших С.А.

Усачева Нина (11А класс) — Призер, Учитель — Меньших С.А.

Литература 

Низовибатько Вероника (9Б класс) — Призер, Учитель — Баранов А.Н.

Русский язык

Дорожкина Анастасия (9А класс) — Призер, Учитель — Айрапетова С.Г.

Информатика

Закурнаев Григорий (9А класс) — Призер, Учитель — Скворцов А.А., Поветьев А.Ю.

Яковлева Наталья (9А класс) — Призер, Учитель — Скворцов А.А.

Бергман Валерий (10А класс) — Призер, Учитель — Скворцов А.А., Поветьев А.Ю.

Бураков Андрей (11А класс) — Призер, Учитель — Скворцов А.А., Поветьев А.Ю.

Нехаев Антон (11А класс) — Призер, Учитель — Скворцов А.А., Поветьев А.Ю.

Химия 

Гладышева Варвара (11А класс) — Призер, Учитель — Гусарова Л.В.

Английский язык

Дорожкина Анастасия Станиславовна (9А класс) — Призер, Учитель — Кобзева О. О. 

Семилетов Андрей Игоревич (9Б класс) — Призер, Учитель — Кобзева О. О. 

Сергеева Анна Владимировна (9Б класс) -Призер, Учитель — Кобзева О. О.

Ефименко Сергей Юрьевич (10Б класс) — Призер, Учитель — Нестерова С. А.

Киселева Анна Юрьевна (10А класс) — Призер, Учитель — Нестерова С. А.

Полянская Светлана Дмитриевна (11Б класс) — Призер, Учитель — Остражкова Н. С.

Пустовалов Иван Андреевич (11Б класс) — Призер, Учитель — Остражкова Н. С.

Чуксина Надежда Алексеевна (11Б класс) — Призер, Учитель — Остражкова Н. С.

Физическая культура

Вальдман Игорь Петрович (10А класс) — Победитель, Учитель — Морковин И. С.

Результаты школьного этапа

Результаты муниципального этапа

 


 

Результаты ВОШ — 2019/2020 учебный год

Дмитриевцев Юрий (11М) — призер заключительного этапа олимпиады ВОШ по астрономии (учитель — С.А. Меньших)

Дмитриевцев Юрий (11М) — призер заключительного этапа олимпиады ВОШ по физике (учитель — В.И. Якунин)

Результаты регионального этапа

Результаты муниципального этапа

Результаты школьного этапа


 

Результаты ВОШ — 2018/2019 учебный год

Дмитриевцев Юрий (10М) — призер заключительного этапа олимпиады ВОШ по астрономии (учитель — С.А. Меньших)

Результаты регионального этапа

Результаты муниципального этапа

Результаты школьного этапа

 

Результаты ВОШ — 2017/2018 учебный год:

Хохлова Анастасия (9фм), Козодаева Валерия (10фм)  — призеры заключительного этапа олимпиады ВОШ по ОБЖ (учитель — Ю.В. Горелкин)

Результаты регионального этапа ВОШ — 2017/2018 учебный год

Результаты муниципального этапа ВОШ — 2017/2018 учебного год

Результаты школьного этапа ВОШ — 2017/2018 учебный год

 

Результаты ВОШ — 2016/2017 учебныйгод:

Федько Василий  — победитель заключительного этапа олимпиады ВОШ по ОБЖ (учитель — Ю.В. Горелкин)

Результаты регионального этапа ВОШ

Результаты муниципального этапа

 

MIL-OSI Россия: Подведены итоги пробного этапа Всероссийской олимпиады школьников

Сообщение спонсируется NewzEngine.com

MIL OSI Translation. Регион: Российская Федерация —

В нем приняли участие более 235 тысяч учащихся из 550 школ Москвы.

В столице подведены итоги пробного этапа Всероссийской олимпиады школьников. По его результатам более 130 тысяч детей будут отмечены почетными грамотами.

Конкурс проходил онлайн с 13 апреля по 25 мая. В нем приняли участие 235 тысяч учеников из 550 столичных школ. Для учащихся 4-10 классов разработаны задания по 24 предметам. Однако соревнования по русскому языку и математике проводились среди учащихся 3-10 классов, по искусству (MHC), физике и экономике — в 6-10 классах, а по химии — в 7-10 классах.

На иностранных языках — английском, немецком, испанском, китайском и французском — включены аудио и видео файлы для школьников.

По словам куратора Центра повышения квалификации Ивана Ященко, в этом году в пробном этапе олимпиады участвовало рекордное количество детей сразу по трем и более предметам.

«Таких ребят было более 57 тысяч. Кроме того, следует отметить, что многие школьники благодаря пробному этапу впервые приняли участие во Всероссийской олимпиаде. В этом году более 51 тысячи студентов ознакомились с конкурсом и заинтересовались им », — сказал он.

Иван Ященко пригласил школьников осенью приобщиться к школьному этапу олимпиады.

Всероссийская олимпиада школьников проводится ежегодно для учащихся 4-11 классов. Конкурс разделен на четыре этапа: школьный, муниципальный, региональный и финальный. Победители и призеры финала могут поступать в вузы России без экзаменов на специальности, соответствующие профилю олимпиады. Организатором пробного этапа олимпиады выступил Центр педагогического мастерства.Задачи и решения можно посмотреть на странице конкурсов.

ПРИМЕЧАНИЕ РЕДАКТОРА. Эта статья является переводом. Приносим извинения, если грамматика и / или структура предложения не идеальны.

MIL OSI Новости России —

заданий олимпиады по физике для учителей. Коллектив лаборатории награжден правительственной наградой

.

Выберите документ из архива для просмотра:

Методические рекомендации по проведению и оценке школьного этапа олимпиады.docx

Библиотека
материалов

    На школьном этапе рекомендуется включить в задание 4 задания для учащихся 7 и 8 классов. Выделить на их выполнение 2 часа; для учащихся 9, 10 и 11 классов — по 5 заданий, на которые следует выделить 3 часа.

    Задания каждой возрастной параллели составлены в одной версии, поэтому участники должны сидеть по одному за стол (парту).

    Перед началом раунда участник заполняет обложку тетради, указывая на ней свои данные.

    Участники завершают свою работу синими или фиолетовыми перьями. Не используйте ручки с красными или зелеными чернилами для написания решений.

    Во время олимпиады допускается использование простого инженерного калькулятора участниками олимпиады. И наоборот, недопустимо использовать справочники, учебники и т. Д. При необходимости студентам должны быть предоставлены таблицы Менделеева.

Система оценки результатов олимпиады

    Количество баллов за каждое задание теоретическое тур находится в диапазоне от 0 до 10 баллов.

    Если проблема решена частично, то этапы решения проблемы подлежат оценке. Не рекомендуется вводить дробные точки. В крайнем случае, их следует округлить «в пользу учащегося» до целых баллов.

    Не разрешается вычитать баллы за «плохой почерк», неаккуратные записи или за решение проблемы способом, не совпадающим с методом, предложенным методической комиссией.

Примечание. В общем, не стоит слишком догматично следовать авторской системе оценок (это всего лишь рекомендации!).Решения и подходы школьников могут отличаться от авторских и не быть рациональными.

    Особое внимание следует уделить прикладному математическому аппарату, применяемому для задач, не имеющих альтернативных решений.

Пример соответствия между набранными баллами и решением, данным участником олимпиады

баллов

Правильность (ошибочность) решения

Полное правильное решение

Правильное решение.Есть несколько мелких ошибок, которые обычно не влияют на решение.

Выбранный документ для просмотра Школьный этап олимпиады по физике 9 класс.docx

Библиотека
материалов

Оценка 9

1. Поезд движения.

т 1 = 23 в т 2 = 13 c

2. Расчет электрических схем.

R 1 знак равно R 4 = 600 Ом, R 2 знак равно R 3 = 1.8 кОм.

3. Калориметр.

т 0 , 0 O С . M , его удельная теплоемкость с , г. λ м .

4. Цветные очки.

5.Колбу в воде.

3 вместимостью 1,5 литра имеет массу 250 г. Вес, какую массу нужно поместить в колбу, чтобы она утонула в воде? Плотность воды 1 г / см 3 .

1. Экспериментатор Глюк наблюдал встречное движение скоростного поезда и электропоезда. Оказалось, что каждый из поездов проезжал мимо Глюка одновременно. т 1 = 23 в … Тем временем друг Глюка, теоретик Баг, был в поезде и определил, что скорый поезд проехал мимо него за т 2 = 13 c… Во сколько раз отличается длина поезда и электрички?

Решение.

Критерии оценки:

    Написание уравнения движения скоростного поезда — 1 балл

    Написание уравнения движения электропоезда — 1 балл

    Написание уравнения движения при приближении скоростного поезда и электропоезда — 2 балла

    Решение уравнения движения, написание формулы в общем виде — 5 баллов

    Математические вычисления -1 балл

2. Какое сопротивление цепи при разомкнутом и замкнутом переключателе? R 1 знак равно R 4 = 600 Ом, R 2 знак равно R 3 = 1,8 кОм.

Решение.

    При открытом ключе: R o = 1,2 кОм.

    С закрытым ключом: R o = 0,9 кОм

Эквивалентная схема с замкнутым ключом:

Критерии оценки:

    Нахождение полного сопротивления цепи с разомкнутым ключом — 3 балла

    Эквивалентная схема с замкнутым ключом — 2 балла

    Нахождение полного сопротивления цепи с замкнутым ключом — 3 балла

    Математические вычисления, перевод единиц — 2 балла

3. В калориметр с водой, температура которой т 0 , бросил кусок льда, который имел температуру 0 O С . После установления теплового равновесия выяснилось, что четверть льда не растаяла. Учитывая известную массу воды M , его удельная теплоемкость с , г. удельная теплота плавления льда λ , найти начальную массу ледяной глыбы м .

Решение.

Критерии оценки:

    Составление уравнения количества тепла, отдаваемого холодной водой — 2 балла

    Решение уравнения теплового баланса (написание формулы в общем виде, без промежуточных расчетов) — 3 балла

    Заключение единиц измерения для проверки формулы расчета — 1 балл

4. На тетради красным карандашом написано «отлично» и «зеленым» — «хорошо».Есть два стакана — зеленый и красный. Через какое стекло нужно посмотреть, чтобы увидеть слово «отлично»? Поясните свой ответ.

Решение.

    Если красное стекло поднести к записи красным карандашом, то оно не будет видно, потому что красное стекло пропускает только красные лучи, а весь фон будет красным.

    Если посмотреть на надпись красным карандашом через зеленое стекло, то на зеленом фоне мы увидим слово «отлично», написанное черными буквами, т.к.зеленое стекло блокирует красные световые лучи.

    Чтобы увидеть в блокноте слово «отлично», нужно посмотреть через зеленое стекло.

Критерии оценки:

5. Колба стеклянная плотностью 2,5 г / см 3 вместимостью 1,5 литра имеет массу 250 г. Какую массу нужно поместить в колбу, чтобы она погрузилась в воду? Плотность воды 1 г / см 3 .

Решение.

Критерии оценки:

    Написание формулы для нахождения силы тяжести, действующей на колбу с грузом — 2 балла

    Написание формулы для нахождения силы Архимеда, действующей на погруженную в воду колбу — 3 балла

Выбранный документ для просмотра Школьный этап олимпиады по физике 8 класс.docx

Библиотека
материалов

Школьный этап олимпиады по физике.

8 класс

    Путешественник.

    Попугай Кеша.

В то утро попугай Кешка, как обычно, собирался сделать доклад о пользе выращивания бананов и поедания бананов. Позавтракав 5 бананами, он взял мегафон и поднялся на «трибуну» — на вершину пальмы высотой 20 метров.На полпути он почувствовал, что не сможет добраться до вершины с мегафоном. Потом оставил мегафон и дальше полез без него. Сможет ли Кешка сделать отчет, если для отчета нужен запас энергии 200 Дж, один съеденный банан позволяет делать работу 200 Дж, масса попугая 3 кг, масса мегафона 1 кг? (при расчете acceptg = 10 Н / кг)

    Температура.

О

    Льдина.

плотность льда

Ответы, инструкции, решения олимпиадных задач

1. Путешественник ехал 1 час 30 минут на верблюде со скоростью 10 км / час, затем 3 часа на осле со скоростью 16 км / час. Какая была средняя скорость путешественника в пути?

Решение.

Критерии оценки:

    Написание формулы средней скорости движения — 1 балл

    Нахождение пройденного расстояния на первом этапе движения — 1 балл

    Нахождение пройденного расстояния на втором этапе движения — 1 балл

    Математические вычисления, перевод единиц — 2 балла

2. В то утро попугай Кешка, как обычно, собирался сделать доклад о пользе выращивания бананов и поедания бананов. Позавтракав пятью бананами, он взял мегафон и поднялся на «трибуну» — верхушку пальмы высотой 20 метров. На полпути он почувствовал, что не может добраться до вершины с мегафоном. Потом оставил мегафон и дальше полез без него. Сможет ли Кешка сделать отчет, если для отчета нужен запас энергии в 200 Дж, один съеденный банан позволяет сделать работу в 200 Дж, масса попугая 3 кг, масса мегафона 1 кг?

Решение.

Критерии оценки:

    Определение общего запаса энергии от съеденных бананов — 1 балл

    Энергия, затраченная на подъем тела на высоту h — 2 балла

    Энергия, затраченная Кешкой на подиум и выступление — 1 балл

    Математические расчеты, правильная формулировка окончательного ответа — 1 балл

3. В воду массой 1 кг, температура которой 10 ° С, влить 800 г кипятка.Какая будет конечная температура смеси? Удельная теплоемкость воды

Раствор.

Критерии оценки:

    Составление уравнения количества тепла, получаемого холодной водой — 1 балл

    Составление уравнения количества тепла, отдаваемого горячей водой — 1 балл

    Запись уравнения теплового баланса — 2 балла

    Решение уравнения теплового баланса (написание формулы в общем виде, без промежуточных вычислений) — 5 баллов

4. По реке плывет плоская льдина толщиной 0,3 м. Какова высота выступающей над водой части льда? Плотность водяного льда

Раствор.

Критерии оценки:

    Учет условий плавания тел — 1 балл

    Написание формулы для нахождения силы тяжести, действующей на льдину — 2 балла

    Написание формулы для нахождения силы Архимеда, действующей на льдину в воде — 3 балла

    Решение системы двух уравнений — 3 балла

    Математические вычисления — 1 балл

Выбранный документ для просмотра Школьный этап олимпиады по физике 10 класс.docx

Библиотека
материалов

Школьный этап олимпиады по физике.

Марка 10

1. Средняя скорость.

2. Эскалатор.

Эскалатор метро поднимает стоящего на нем пассажира за 1 минуту. Если человек идет по остановившемуся эскалатору, подъем займет 3 минуты. Сколько времени потребуется, чтобы подняться, если человек идет по восходящему эскалатору?

3.Ведерко со льдом.

M с = 4200 Дж / (кг O λ = 340,000 Дж / кг.

т˚, С

т, мин

т, мин минминмин

4. Эквивалентная схема.

Найдите сопротивление цепи, показанной на рисунке.

2

рэнд 2

рэнд 2

рэнд 2

рэнд 2

рэнд 2

рэнд -?

5.Баллистический маятник.

м

Ответы, инструкции, решения олимпиадных задач

1 . Путешественник путешествовал из города А в город Б сначала поездом, а затем верблюдом. Какова средняя скорость путешественника, если он проехал две трети пути на поезде и одну треть пути на верблюде? Скорость поезда 90 км / ч, скорость верблюда 15 км / ч.

Решение.

    Обозначим расстояние между точками через s.

Тогда время в пути поездом:

Критерии оценки:

    Написание формулы для определения времени на первом этапе пути — 1 балл

    Написание формулы нахождения времени на втором этапе движения — 1 балл

    Нахождение всего времени движения — 3 балла

    Вывод формулы расчета для нахождения средней скорости (написание формулы в общем виде, без промежуточных вычислений) — 3 балла

    Математические вычисления — 2 балла.

2. Эскалатор метро поднимает стоящего на нем пассажира за 1 мин. Если человек идет по остановившемуся эскалатору, подъем займет 3 минуты. Сколько времени потребуется, чтобы подняться, если человек идет по восходящему эскалатору?

Решение.

Критерии оценки:

    Составление уравнения движения пассажира на движущемся эскалаторе — 1 балл

    Составление уравнения движения пассажира, движущегося на стационарном эскалаторе — 1 балл

    Составление уравнения движения движущегося пассажира на движущемся эскалаторе — 2 балла

    Решение системы уравнений, определение времени в пути движущегося пассажира на движущемся эскалаторе (вывод формулы расчета в общем виде без промежуточных расчетов) — 4 балла

    Математические вычисления — 1 балл

3. В ведре находится смесь воды и льда общей массой M = 10 кг. Ведро переносили в комнату и сразу же измеряли температуру смеси. Полученная зависимость температуры от времени показана на рисунке. Удельная теплоемкость воды с = 4200 Дж / (кг О С). Удельная теплота плавления льда λ = 340,000 Дж / кг. Определите массу льда в ведре, когда его принесли в комнату.Не обращайте внимания на теплоемкость ведра.

т˚, ˚ С

т, мин мин мин мин

Решение.

Критерии оценки:

    Составление уравнения количества тепла, получаемого водой — 2 балла

    Составление уравнения количества тепла, необходимого для плавления льда — 3 балла

    Запись уравнения теплового баланса — 1 балл

    Решение системы уравнений (написание формулы в общем виде, без промежуточных вычислений) — 3 балла

    Математические вычисления — 1 балл

4. Найдите сопротивление цепи, показанной на рисунке.

2

рэнд 2

рэнд 2

рэнд 2

рэнд 2

рэнд 2

рэнд -?

Решение:

    Два правых сопротивления соединены параллельно и вместе дают R .

    Это сопротивление последовательно соединено с крайним правым сопротивлением со значением R . … Вместе они обеспечивают сопротивление 2 R .

    Таким образом, переходя от правого конца схемы к левому, получаем, что полное сопротивление между входами схемы составляет R .

Критерии оценки:

    Расчет параллельного включения двух резисторов — 2 балла

    Расчет последовательного включения двух резисторов — 2 балла

    Схема эквивалентная — 5 баллов

    Математические вычисления — 1 балл

5. Пуля массой летит горизонтально со скоростью и застревает в ней. На какую высоту H поднимается ящик после попадания пули?

Решение.

Butterfly — 8 км / ч

Fly — 300 м / мин

Cheetah — 112 км / ч

Turtle — 6 м / мин

2. Сокровище.

Обнаружена запись о местонахождении клада: «От старого дуба пройдите на север 20 м, поверните налево и пройдите 30 м, поверните налево и пройдите 60 м, поверните направо и пройдите 15 м, поверните направо и идите пешком. 40 м; копай здесь.«Какой путь, согласно записи, нужно пройти, чтобы добраться от дуба к сокровищу? На каком расстоянии от дуба находится сокровище. Завершите чертеж задания.

3. Таракан Митрофан.

Таракан Митрофан ходит по кухне. Первые 10 с он шел со скоростью 1 см / с в направлении на север, затем повернул на запад и прошел 50 см за 10 с, простоял 5 с, а затем по направлению на северо-восток на скорость 2 см / с, пробег 20 см.Здесь его настигла нога человека. Как долго гулял по кухне таракан Митрофан? Какая средняя скорость передвижения таракана Митрофана?

4. Гонки на эскалаторе.

Ответы, инструкции, решения олимпиадных задач

1. Запишите имена животных в порядке убывания скорости:

    Акула — 500 м / мин

    Бабочка — 8 км / ч

    Полет — 300 м / мин

    Гепард — 112 км / ч

    Черепаха — 6 м / мин

Решение.

Критерии оценки:

    Преобразование скорости бабочки в Международную систему единиц — 1 балл

    Преобразование скорости движения мухи в СИ — 1 балл

    Преобразование скорости передвижения гепарда в СИ — 1 балл

    Преобразование скорости передвижения черепахи в СИ — 1 балл

    Написание названий животных в порядке убывания скорости передвижения — 1 балл.

    • Гепард — 31,1 м / с

      Акула — 500 м / мин

      Муха — 5 м / с

      Бабочка — 2,2 м / с

      Черепаха — 0,1 м / с

2. Была найдена запись о местонахождении клада: «От старого дуба пройдите 20 м на север, поверните налево и пройдите 30 м, поверните налево и пройдите 60 м, поверните направо и пройдите 15 м, поверните направо и пройдите 40 м; копай здесь. «Какой путь, согласно летописи, нужно пройти, чтобы перейти от дуба к сокровищу? На каком расстоянии от дуба находится сокровище.Завершите рисунок задания.

Решение.

Критерии оценки:

    Построение плана траектории в масштабе: 1см 10м — 2 балла

    Нахождение пройденного пути — 1 балл

    Понимание разницы между пройденным расстоянием и движением тела — 2 балла

3. По кухне ходит таракан Митрофан. Первые 10 с он шел со скоростью 1 см / с в направлении на север, затем повернул на запад и прошел 50 см за 10 с, простоял 5 с, а затем по направлению на северо-восток на скорость 2 см / с, пробег 20 см.

Здесь его настигла нога человека. Как долго гулял по кухне таракан Митрофан? Какая средняя скорость передвижения таракана Митрофана?

Решение.

Критерии оценки:

    Определение времени движения на третьем этапе движения: — 1 балл

    Нахождение пройденного пути на первом этапе движения таракана — 1 балл

    Написание формулы нахождения средней скорости передвижения таракана — 2 балла

    Математические вычисления — 1 балл

4. Двое малышей Петя и Вася решили устроить скачки на спускающемся эскалаторе. При этом они побежали из одной точки, расположенной ровно посередине эскалатора, в разные стороны: Петя — вниз, а Вася — вверх по эскалатору. Время, проведенное Васей на дистанции, оказалось в 3 раза больше, чем у Пети. С какой скоростью движется эскалатор, если на прошлых соревнованиях друзья показали такой же результат, пробежав такое же расстояние со скоростью 2,1 м / с?

Найдите материал для любого урока,

олимпиадных задач по физике 10 класс с решением.

Олимпиадные задания по физике. Оценка 10.

В системе, показанной на рисунке, блок массы M может скользить по рельсам без трения.
Груз отводится под углом a от вертикали и отпускается.
Определите массу груза m, если угол a не изменяется при перемещении системы.

Тонкостенный наполненный газом цилиндр массы M, высоты H и площади основания S плавает в воде.
В результате потери герметичности нижней части цилиндра глубина его погружения увеличилась на D H.
Атмосферное давление равно P 0, температура не меняется.
Какое было начальное давление газа в баллоне?

Замкнутая металлическая цепь соединена резьбой с осью центробежной машины и вращается с угловой скоростью w.
В этом случае резьба образует угол a с вертикалью.
Найдите расстояние x от центра тяжести цепи до оси вращения.


Внутри длинной трубки, наполненной воздухом, поршень движется с постоянной скоростью.
В этом случае упругая волна распространяется в трубе со скоростью S = 320 м / с.
Считая падение давления на границе распространения волны равным P = 1000 Па, оцените падение температуры.
Давление в невозмущенном воздухе P 0 = 10 5 Па, температура T 0 = 300 К.

На рисунке показаны два замкнутых процесса с одним и тем же идеальным газом 1-2-3-1 и 3-2-4-2.
Определите, в каком из них газ отлично поработал.


Пусть T — сила натяжения нити, a 1 и a 2 — ускорения тел с массами M и m.


Записывая уравнения движения для каждого из тел вдоль оси x, получаем
a 1 M = T · (1-sina), a 2 m = T · sina.

Поскольку угол a не изменяется во время движения, тогда a 2 = a 1 (1-синус). Легко видеть, что


а 1 а 2
= м (1- сина) Мсина
= 1 1-сина
.

Отсюда

Учитывая вышеизложенное, мы, наконец, находим


P = f
s
и
P 0 + gM S
c
h
NS
f
s
и
1- D H H
c
h
NS
.

Чтобы решить эту проблему, следует отметить, что
, что центр масс цепи вращается по окружности радиуса x.
В этом случае на цепь действуют только сила тяжести, приложенная к центру масс, и сила натяжения нити T.
Очевидно, что центростремительное ускорение может быть обеспечено только горизонтальной составляющей силы натяжения нити.
Следовательно, mw 2 x = Цина.


В вертикальном направлении сумма всех сил, действующих на цепь, равна нулю; тогда mg- Tcosa = 0.

Из полученных уравнений находим ответ

Пусть волна движется в трубе с постоянной скоростью V.
Свяжем это значение с заданным перепадом давления D P и разностью плотностей D r в невозмущенном воздухе и волне.
Перепад давления разгоняет «избыток» воздуха с плотностью D r до скорости V.
Следовательно, в соответствии со вторым законом Ньютона можно записать

Разделив последнее уравнение на уравнение P 0 = R r T 0 / м, получаем


D P P 0
= Д р р
+ Д Т Т 0
.

Поскольку D r = D P / V 2, r = P 0 m / (RT), окончательно находим

Численная оценка с учетом данных, приведенных в постановке задачи, дает ответ D T »0,48K.

Для решения задачи необходимо построить графики круговых процессов в координатах P-V,
, так как площадь под кривой в таких координатах равна работе.
Результат этой конструкции показан на рисунке.


21 февраля в Доме Правительства Российской Федерации состоялась церемония вручения Премий Правительства в области образования за 2018 год.Награды лауреатам вручил заместитель Председателя Правительства Российской Федерации Т.А. Голикова.

Среди лауреатов премии — сотрудники Лаборатории по работе с одаренными детьми. Награду получили преподаватели сборной России по IPhO Виталий Шевченко и Александр Киселев, преподаватели сборной России по IJSO Елена Михайловна Снигирева (химия) и Игорь Киселев (биология) и руководитель сборной России по химии. команда, проректор МФТИ Артем Анатольевич Воронов.

Основные достижения, за которые команда была удостоена правительственной награды — 5 золотых медалей для сборной России на IPhO-2017 в Индонезии и 6 золотых медалей для команды на IJSO-2017 в Голландии. Каждый студент принес домой золото!

Столь высокого результата на Международной олимпиаде по физике российская команда достигла впервые. За всю историю IPhO с 1967 года ни сборной России, ни сборной СССР ни разу не удавалось завоевать пять золотых медалей.

Сложность задач олимпиады и уровень подготовки команд из других стран постоянно растет. Однако сборная России в последние годы входит в пятерку лучших команд мира. Для достижения высоких результатов преподаватели и руководство сборной совершенствуют систему подготовки к стажировке в нашей стране. Появились образовательные школы, где ученики детально изучают самые сложные разделы программы.Активно создается база экспериментальных заданий, выполняя которые ребята готовятся к экспериментальному туру. Удаленная работа ведется регулярно; В течение года подготовки дети получают около десяти теоретических домашних заданий. Большое внимание уделяется качественному переводу условий задач на самой Олимпиаде. Учебные курсы совершенствуются.

Высокие результаты на международных олимпиадах — результат многолетней работы большого количества преподавателей, сотрудников и студентов МФТИ, личных преподавателей по специальностям, а также кропотливого труда самих школьников.Помимо вышеупомянутых лауреатов, огромный вклад в подготовку национальной сборной внесли:

Федор Цыбров (создание заданий на квалификационный сбор)

Алексей Ноян (экспериментальная подготовка сборной, развитие экспериментальной мастерской)

Алексей Алексеев (создание заданий на квалификационные сборы)

Арсений Пикалов (подготовка теоретических материалов и проведение семинаров)

Иван Ерофеев (многолетний опыт работы по всем направлениям)

Александр Артемьев (домашнее задание)

Никита Семенин (создание заданий на квалификационный сбор)

Андрей Песков (разработка и создание экспериментальных установок)

Глеб Кузнецов (экспериментальная тренировка сборной)

Задание на 7 класс

Задание 1.Путешествие Незнайка.

В 4 часа вечера Незнайка проехал мимо километрового столба, на котором было написано 1456 км, а в 7 часов утра мимо столба с надписью 676 км. В какое время Незнайка прибудет на станцию, от которой отсчитывается расстояние?

Задание 2. Термометр.

В некоторых странах, например в США и Канаде, температура измеряется не в градусах Цельсия, а в градусах Фаренгейта. На рисунке изображен такой градусник. Определите значение делений шкалы Цельсия и шкалы Фаренгейта и определите значения температуры.

Задача 3. Озорные очки.

Коля с сестрой Олей начали мыть посуду после ухода гостей. Коля вымыл стаканы и, перевернув, поставил на стол, а Оля вытерла полотенцем, потом убрала в шкаф. Но! .. Вымытые стаканы плотно прилипли к клеенке! Почему?

Задача 4. Персидская пословица.

Персидская пословица гласит: «Запах мускатного ореха невозможно скрыть». К какому физическому явлению относится это высказывание? Объясни ответ.

Задача 5. Верховая езда.

Предпросмотр:

Задания для 8 класса.

Задание 1. Верховая езда.

Путешественник ехал сначала на лошади, а затем на осле. Какую часть пути и какую часть всего времени он ехал на лошади, если средняя скорость путешественника составляла 12 км / ч, скорость езды на лошади была 30 км / ч, а на осле — 6 км. / ч?

Задача 2. Лед в воде.

Задача 3. Подъемник для слонов.

Молодые мастера решили соорудить для зоопарка лифт, на котором слоник весом 3 штуки.Из клетки можно поднять 6 тонн на платформу, расположенную на высоте 10 м. По разработанному проекту подъемник приводится в движение мотором от кофемолки мощностью 100Вт, при этом потери энергии полностью исключены. Сколько времени займет каждое восхождение в этих условиях? Рассмотрим g = 10 м / с 2 .

Проблема 4. Неизвестная жидкость.

В калориметре разные жидкости поочередно нагреваются с помощью одного и того же электронагревателя. На рисунке представлены графики зависимости температуры t жидкостей от времени τ.Известно, что в первом опыте калориметр содержал 1 кг воды, во втором — другое количество воды, а в третьем — 3 кг некоторой жидкости. Какова была масса воды во втором эксперименте? С какой жидкостью был проведен третий эксперимент?

Задача 5. Барометр.

Иногда на шкале барометра делают надписи «Ясно» или «Облачно». Какой из этих рекордов соответствует более высокому давлению? Почему прогнозы барометра не всегда верны? Что предсказывает барометр на вершине высокой горы?

Предпросмотр:

Задачи для 9 класса.

Задача 1.

Обоснуйте ответ.

Цель 2.

Цель 3.

Сосуд с водой с температурой 10 ° C был помещен на электрическую плиту. Через 10 минут вода закипела. Сколько времени нужно, чтобы вода полностью испарилась в сосуде?

Задача 4.

Задача 5.

Лед был брошен в стакан, наполненный водой. Изменится ли уровень воды в стакане при таянии льда? Как изменится уровень воды, если свинцовый шар вморожен в кусок льда? (объем шара считается незначительным по сравнению с объемом льда)

Предпросмотр:

Задачи для 10 класса.

Задача 1.

Человек, стоящий на берегу реки шириной 100 м, хочет перейти на другую сторону, на противоположную точку. Он может сделать это двумя способами:

  1. Все время плыть под углом к ​​течению, чтобы результирующая скорость всегда была перпендикулярна берегу;
  2. Плывите прямо к противоположному берегу, а затем пройдите расстояние, на которое его унесет течение. Каким путем вы быстрее перейдете? Он плывет со скоростью 4 км / ч, а идет со скоростью 6.4 км / ч, скорость по реке 3 км / ч.

Цель 2.

В калориметре разные жидкости поочередно нагреваются с помощью одного и того же электронагревателя. На рисунке представлены графики зависимости температуры жидкости t от времени τ. Известно, что в первом опыте калориметр содержал 1 кг воды, во втором — другое количество воды, а в третьем — 3 кг некоторой жидкости. Какова была масса воды во втором эксперименте? С какой жидкостью был проведен третий эксперимент?

Цель 3.

Тело, имеющее начальную скорость V 0 = 1 м / с, двигался равномерно ускоренно и, пройдя некоторое расстояние, приобрел скорость V = 7 м / с. Какая скорость была у тела на половине этого расстояния?

Задача 4.

Две лампочки читают «220В, 60Вт» и «220В, 40Вт». Какая сила тока в каждой из лампочек при последовательном и параллельном подключении, если напряжение в сети 220В?

Задача 5.

Лед бросили в стакан с водой.Изменится ли уровень воды в стакане при таянии льда? Как изменится уровень воды, если свинцовый шар вморожен в кусок льда? (объем мяча считается незначительным по сравнению с объемом льда).

Цель 3.

Три одинаковых заряда q расположены на одной прямой, на расстоянии l друг от друга. Какова потенциальная энергия системы?

Задание 4.

Груз m 1 подвешен на пружине жесткости k и находится в состоянии равновесия.В результате неупругого попадания пули, летящей вертикально вверх, груз начал двигаться и остановился в положении, когда пружина была не растянута (и не сжата). Определить скорость пули, если ее масса составляет м 2. … Не обращайте внимания на массу пружины.

Задача 5.

Лед бросили в стакан с водой. Изменится ли уровень воды в стакане при таянии льда? Как изменится уровень воды, если свинцовый шар вморожен в кусок льда? (объем мяча считается незначительным по сравнению с объемом льда).

по движению за первые 3 секунды движения

8 класс


XLVI Всероссийская физическая олимпиада школьников. Ленинградская обл. Коммунальная сцена

9 класс

     = 2,7 10 3 кг / м 3,  в = 10 3 кг / м 3 и  B = 0,7 10 3 кг / м 3 … Плавучесть воздуха без учета г = 10 м / с 2.

    с = 4.2 кДж / К?

XLVI Всероссийская физическая олимпиада школьников. Ленинградская обл. Коммунальная сцена

10 класс

H H равно V .

Q


4
ρ ρ v … Определить отношение ρ / ρ v … Ускорение свободного падения g .

XLVI Всероссийская физическая олимпиада школьников.Ленинградская обл. Коммунальная сцена

11 класс

v . R г .

3. Какой максимальный объем воды с плотностью ρ 1 = 1,0 г / см 3 можно заливать в Н-образную асимметричную трубку с открытыми верхними концами, частично заполненную маслом плотностью ρ 2 = 0,75 г / см 3? Площадь горизонтального сечения вертикальных частей трубы составляет S . … Объемом горизонтальной части трубки можно пренебречь.Вертикальные размеры трубы и высота масляного столба показаны на рисунке (высота х ). считается данным).

Примечание.

4. Каково сопротивление проволочного каркаса в форме прямоугольника со сторонами , но не и , и диагональю, если ток течет из точки A в точку B? Сопротивление на единицу длины провода .

    Движение материальной точки описывается уравнением x (t) = 0.2 sin (3.14t), где x выражается в метрах, t — в секундах. Определите путь, пройденный точкой за 10 секунд движения.

Возможные решения

7 класс

    На графике показана зависимость пройденного телом пути от времени. Какой из графиков соответствует зависимости скорости этого тела от времени?

Решение: Правильный ответ — G.

2. Из абзаца

A к точке B Автомобиль «Волга» выехал на скорости 90 км / ч.При этом навстречу ему с точки B машина «Жигули» уехала. В 12 часов дня машины проезжали мимо друг друга. В 12:49 Волга прибыла на точку Б . , а через 51 минуту «Жигули» прибыли на A … Посчитайте скорость Жигулей.

Решение: «Волга» проехала от точки А до места встречи с «Жигулями» за т. x , а «Жигули» проехали на одном участке т. 1 = 100 минут.В свою очередь «Жигули» проехали от точки Б до места встречи с Волгой за т. . x , а «Волга» на этом же участке проехала т. 2 = 49 минут. Запишем эти факты в виде уравнений:

где υ 1 — скорость «Жигулей», а υ 2 — скорость Волги. Разделив один член уравнения на другой, получим:


.

Отсюда υ 1 = 0,7 υ 2 = 63 км / ч.

3. Материальная точка движется по окружности радиусом R = 2 м с постоянным модулем скорости, совершая полный оборот за 4 с. Определить среднюю скорость по перемещению за первые 3 секунды движения

Решение: Перемещение материальной точки за 3 секунды составляет

Средняя скорость движения составляет
/3

4. Тело движется так, что его скорости в каждый из n равных интервалов времени равны V 1, V 2, V 3,… ..V n соответственно.Какая средняя скорость тела?

Решение:

XLVI Всероссийская физическая олимпиада школьников. Ленинградская обл. Коммунальная очередь

Возможные решения

8 класс


Раствор: F 1 мг = F 1 + F 2 F 2

 3 гВ = 1 гВ 2/3 +  2 гВ 1/3

мг 3 =  1 2/3 +  2 1/3

 3 = (2  1 +  2) / 3

2. Междугородний автобус преодолел 80 км за 1 час.Двигатель развивал мощность 70 кВт при КПД 25%. Сколько дизельного топлива (плотность 800 кг / м 3, удельная теплота сгорания 42 10 6 Дж / кг) сэкономил водитель при норме расхода топлива 40 л на 100 км пути?

Решение: КПД = A / Q = Nt / rm = Nt / r V

V = Nt / r  КПД

Расчеты: V = 0,03 м 3; из соотношения 80/100 = x / 40 определяем норму расхода топлива на 80 км x = 32 (литров)

V = 32-30 = 2 (литров)

3.Человека перевозят на лодке из точки А в точку Б, расположенную на кратчайшем расстоянии от точки А с другой стороны. Скорость лодки относительно воды 2,5 м / с, скорость реки 1,5 м / с. За какое минимальное время он переправится, если ширина реки 800 м?

Решение: Для перехода за минимальное время необходимо, чтобы вектор результирующей скорости v был направлен перпендикулярно берегу

4. Тело проходит одни и те же участки пути с постоянными скоростями в пределах раздел V 1, V 2, V 3,….. В п. Определите среднюю скорость на всем пути.

Решение:

XLVI Всероссийская физическая олимпиада школьников. Ленинградская обл. Коммунальная очередь

Возможные решения

9 класс

    Полый алюминиевый шар, находясь в воде, растягивает пружину динамометра с силой 0,24 Н, а в бензине с силой 0,33 Н. Найдите объем полости. Плотности алюминия, воды и бензина соответственно = 2.7 10 3 кг / м 3, дюйма = 10 3 кг / м 3 и  B = 0,7 10 3 кг / м 3 г = 10 м / с 2.

Решение:

R раствор: Куб находится в равновесии под действием трех сил: силы тяжести м г , Архимедова сила F A и сила реакции со стороны опор, которую, в свою очередь, удобно разложить на две составляющие: составляющую силы реакции, перпендикулярную наклонному дну N и сила трения на опорах F тр.

Обратите внимание, что наличие опор, на которых опирается куб, играет важную роль в проблеме, поскольку именно благодаря им вода окружает куб со всех сторон, и для определения силы, с которой вода действует на него, вы можете использовать закон Архимеда. Если бы куб лежал прямо на дне сосуда и вода не просачивалась под ним, то возникающие поверхностные силы давления воды на куб не толкали бы его вверх, а, наоборот, еще сильнее прижимали бы к нему. Нижний.В нашем случае на куб действует выталкивающая сила F A = a 3 g , направленная вверх.

Проецируя все силы на координатную ось, параллельную дну сосуда, запишем условие равновесия куба в виде: F tr = ( mg — F A) sin.

Учитывая, что масса куба m =  a a 3, получаем ответ: F tr = ( a дюйм ) a 3 g sin = 8.5 (Н).

    Камень, брошенный под углом  30 0 к горизонту, дважды находился на одинаковой высоте h; через время t 1 = 3 с и время t 2 = 5 с после начала движения. Найдите начальную скорость тела. Ускорение свободного падения Земли составляет 9,81 м / с 2.

Решение: Движение тела в вертикальном направлении описывается уравнением:

Отсюда при y = h получаем;

Используя свойства корней квадратного уравнения, согласно которым

получаем

    Ускорение свободного падения на поверхности Солнца равно 264.6 м / с 2, а радиус Солнца в 108 раз больше радиуса Земли. Определите соотношение плотностей Земли и Солнца. Ускорение свободного падения Земли составляет 9,81 м / с 2.

Решение: Мы применяем закон всемирного тяготения, чтобы определить г

    Для измерения температуры 66 г воды в нее погружали термометр, имеющий теплоемкость C T = 1,9 Дж / К, который показывал температуру в помещении t 2 = 17.8 0 C. Какова фактическая температура воды, если термометр показывает 32,4 0 C. Теплоемкость воды с = 4,2 кДж / К?

Раствор: Термометр при погружении в воду получил количество тепла
.

Это количество тепла дает ему вода; Следовательно,
.

Отсюда

XLVI Всероссийская физическая олимпиада школьников. Ленинградская обл. Коммунальная очередь

Возможные решения

Марка 10

1.Воздушный пузырек поднимается со дна резервуара глубиной H … Найти зависимость радиуса воздушного пузыря от глубины его положения в текущий момент времени, если его объем на глубине H равен до В .

Раствор: Давление на дне резервуара:
на глубине ч :

Объем пузыря на глубине ч :

Отсюда

2. За время t 1 = За 40 с в цепи, состоящей из трех одинаковых проводников, соединенных параллельно и подключенных к сети, выделилось определенное количество тепла Q … Сколько времени потребуется, чтобы выделить такое же количество тепла, если проводники соединены последовательно?

Решение:

3. Можно ли подключить последовательно две лампы накаливания мощностью 60 Вт и 100 Вт, рассчитанные на напряжение 110 В, в сеть с напряжением 220 В, если разрешено напряжение на каждой лампе? превышать не более 10% от номинала? Вольт-амперная характеристика (зависимость тока в лампе от приложенного напряжения) представлена ​​на рисунке.

Решение: При номинальном напряжении U n = 110 В ток, протекающий через лампу мощностью P 1 = 60 Вт, составляет
А. При последовательном соединении ламп будет идти такой же ток. через лампу мощностью Р2 = 100 Вт. По вольт-амперной характеристике этой лампы при токе 0,5 А напряжение на этой лампе должно быть
В. Следовательно, при последовательном соединении двух ламп, напряжение на лампе мощностью 60 Вт достигает номинального уже при сетевом напряжении
В.Следовательно, при напряжении 220 В напряжение на этой лампе будет превышать номинальное более чем на 10%, и лампа перегорит.

4
… Два одинаковых шара плотностью ρ связаны невесомой нитью, перекинутой через блок. Правый шар погружен в вязкую жидкость плотностью ρ 0, поднимается с постоянной скоростью v … Определить положение ρ / ρ 0, если установившаяся скорость шара, свободно падающего в жидкость, также равна v … Ускорение свободного падения г .

Решение: Силы сопротивления движению шариков из-за равенства их установившихся скоростей в обоих случаях одинаковы, хотя и направлены в противоположные стороны.

Запишем динамическое уравнение движения в проекциях на ось OU , направленную вертикально вверх для первого и второго случаев (движение системы тел и падение одного шара в жидкость соответственно):

Т — мг = 0

T + F A — мг — F c = 0

F A — мг + F c = 0,

где мг — модуль силы тяжести, T — модуль силы натяжения нити, F A — модуль плавучести, F c — модуль силы сопротивления.

Решая систему уравнений, получаем
.

5. Спортсмены бегут с одинаковой скоростью v с длиной колонны l 0. Тренер бежит навстречу скорости u (u Возможные решения

11 класс

1. Колесо радиуса R катится без пробуксовки с постоянной скоростью центра колеса v … Камешек падает с верха обода колеса. Сколько времени потребуется колесу, чтобы ударить по этой гальке? Радиус колеса R , ускорение свободного падения г .

Решение: Если ось колеса движется со скоростью v, без пробуксовки, то скорость нижней точки равна 0, а верхней, как и горизонтальной скорости камешка, 2 v .

Время падения камешка

Время перемещения горизонтальной оси
вдвое больше.

Это означает, что столкновение произойдет в
.

2. Муравей убегает от муравейника по прямой, так что его скорость обратно пропорциональна расстоянию до центра муравейника.В момент, когда муравей находится в точке A на расстоянии l 1 = 1 м от центра муравейника, его скорость v 1 = 2 см / с. Сколько времени нужно муравью, чтобы пробежать от точки A до точки B, которая находится на расстоянии l 2 = 2 м от центра муравейника?

Решение: Скорость муравья не изменяется линейно со временем. Следовательно, средняя скорость на разных участках пути разная, и мы не можем использовать известные формулы для средней скорости для ее решения.Разобьем путь муравья от точки A до точки B на небольшие участки, пройденные за равные промежутки времени
… Тогда ρ 2 = 0,75 г / см 3? Площадь горизонтального сечения вертикальных частей трубы S … Объемом горизонтальной части трубы можно пренебречь. Вертикальные размеры трубы и высота масляного столба показаны на рисунке (высота х считается заданной).

Примечание. Запрещается заглушать открытые концы трубки, наклонять ее или выливать из нее масло.

Решение: Важно, чтобы в коротком колене оставалось как можно меньше масла. Тогда в высокой трубе можно будет создать столб с максимальной высотой более 4 х на NS … Для этого начните заливать воду в правое колено. Так будет продолжаться до тех пор, пока уровень воды не достигнет 2 х в правом колене, а уровень масла соответственно не составит 3 х в левом колене. Дальнейшее вытеснение масла невозможно, так как граница раздела масло-вода в правом колене станет выше, чем соединительная труба, и вода начнет поступать в левое колено.Процесс добавления воды необходимо остановить, когда верхняя часть масла в правом колене достигнет верхней части колена. Условие равенства давлений на уровне соединительной трубки дает:

5. Движение материальной точки описывается уравнением x (t) = 0,2 sin (3,14t), где x выражается в метрах, t — в секундах. Определите путь, пройденный точкой за 10 секунд движения.

Решение: Движение описывается уравнением:

;

отсюда T = 1 с. За 10 с точка совершит 10 полных колебаний.За одно полное колебание точка проходит путь, равный 4 амплитудам.

Полный путь 10x 4x 0,2 = 8 м

Это настоящий спорт: Москвичи

На этой неделе произошла настоящая сенсация: российские школьники установили абсолютный рекорд XXI Азиатской олимпиады по физике — все восемь участников национальной сборной завоевали золотые медали. В составе сборной России четверо москвичей — их уже поздравил Сергей Собянин.

О том, как готовились москвичи и с чем им пришлось столкнуться на физической олимпиаде — в мос.RU. материал.

Тяжелая теория

Азиатская олимпиада по физике проводится с 2000 года. За это время она стала одним из самых престижных международных соревнований в своей дисциплине, а московские призеры называют его стандартным комплексным. В этом году за победу боролись более 180 участников из 23 стран — правда, в нестандартном формате: из-за пандемии этапы проводились онлайн.

Школьники проверяли свои знания в двух турах: теоретическом и экспериментальном, который из-за ограничений превратился в виртуальный.Все физические процессы и оборудование для экспериментов моделировались компьютерной программой, которую использовали участники олимпиады. Темой экспериментов на этом этапе была оптика: студенты анализировали задачи с дифракцией и эксперименты с кантилевером — устройством в атомно-силовом микроскопе для измерения мельчайших возмущений.

«С одной стороны, было легче соревноваться дистанционно, потому что на Тайване, куда нам пришлось бы ехать, климат и окружающая среда отличаются от наших, нет знакомых лиц.Но, с другой стороны, этот формат накладывал ограничения: например, перевод задач на русский язык ограничивался пятью часами. Иногда приходилось смотреть где-то в английской версии, проверять оригинал, чтобы точно понять задачу, — рассказывает Данила Самоделкин, ученик физико-математической школы № 2007 г. Южное Бутово.

Одной из самых серьезных проблем на пути к золотой медали был не языковой барьер и даже не сложное компьютерное моделирование экспериментов, а теоретическая часть.По словам руководителя группы Юрия Скакова, математически она оказалась загруженной. Учащийся школы № 57 Николай Кононенко добавил, что задачи теоретического тура были сложными как с точки зрения идей, так и с точки зрения решения.

«Было три задачи по таким сложным темам, как гидравлический удар, трассировка лучей и магнитная левитация — подъем объектов с использованием только магнитного поля. Все они состояли из большого количества элементов и их было очень трудно решить, потому что они не были достаточно обозначены и не содержат руководящих идей », — сказал Николай.

Другие обладатели золотых медалей также отметили сложность теоретического этапа. Так, Артемий Новиков, ученик Инженерного училища № 1580 им. Баумана, сказал, что это существенно повлияло на результаты всех участников.

«Задания оказались намного сложнее, чем на предыдущих олимпиадах — это видно по среднему баллу. Обычно каждый набирает 20-30 баллов из 30, а в этом году почти вся сборная страны набрала 10 баллов. .А вот остальные участники набрали в основном либо два, либо три, средний балл заметно ниже 10 », — добавил медалист.

Годовое обучение

Рецепт успеха национальной сборной довольно прост: большой объем подготовительной работы, кропотливая подготовка и продуманное планирование. Однако и здесь есть несколько хитростей.

Особенности подготовки сборной России помогли ей справиться с трудностями теоретического этапа.Ребята отмечают, что их команда отличается высоким уровнем квалификации и способностью глубже разбираться в сложных проблемах.

«В олимпиаде принимают участие азиатские страны, которые традиционно отличаются тем, что умеют не ошибаться, то есть очень хорошо выполняют механическую работу. У нас механическая работа хуже, но креативность лучше. Это Оказывается, сложный теоретический тур был бонусом для российской команды, которая может побеждать, развивая идеи », — сказал Данила Самоделкин.

Руководитель группы Юрий Скаков также подтверждает, что уровень подготовки ребят заметно повысился. Кроме того, была изменена программа тренировок. Наставники выделили самые важные темы, и многие из них выпали ребятам на олимпиаде. Это помогло национальной команде хорошо выступить.

Более того, студенты сами поддерживали и мотивировали друг друга двигаться дальше, несмотря на внутрикомандное соперничество.

«Подготовка оказалась дружественной.За этот год ребята сформировали отличный коллектив, хотя везде выступают лично. Мы подталкивали друг друга, вдохновляли друг друга не сдаваться », — добавил тренер.

Сборная страны готовилась к олимпиаде почти год — с августа прошлого года. Ради достижения цели школы даже составили индивидуальную программу для участников олимпиады. Кроме того, будущие обладатели золотых медалей регулярно выезжали на сборы, не забывая при этом о постоянной домашней работе — самостоятельном решении задач международных олимпиад прошлых лет.

«Мы также участвовали в тренировочных олимпиадах, например, в Международной Жаутыковской олимпиаде, Олимпиаде мегаполисов, Всероссийской олимпиаде школьников — это тоже часть подготовки. Требуется полная самоотдача, а самоподготовка очень важна. Кроме того, необходимо изучать новый материал на сборах, а эта программа — это вовсе не школьная, а уже университетская программа, и для передачи этих знаний ученикам требуется высокая квалификация учителей », — Николай Кононенко. сказал.

На пути к вершине

Если вы спросите кого-нибудь из финалистов, что их ждет после Азиатской олимпиады по физике, ответ будет тот же — Международная физическая олимпиада школьников, или IPhO (Международная олимпиада по физике).

Каждый из победителей хочет участвовать в самом престижном физическом соревновании — из восьми обладателей золотых медалей только пятеро попадут на последнюю строчку соревнований. Кто именно будет представлять Россию, решит тренерский штаб, и особенно будет учтено участие в Азиатской физической олимпиаде.Кстати, московские победители IPhO получат не только награды и медали — Правительство города поощряет их денежными призами.

Артемий Новиков подчеркивает, что общая цель для команды выше соперничества и все участники остаются друзьями. При этом медалист отмечает, что участие в IPhO — это практически спортивный интерес, напрямую не связанный с дальнейшими планами на жизнь. Сейчас Артемий интересуется гидродинамикой.

«Олимпиада и научная физика — разные вещи.Олимпиада по физике — это настоящий спорт. Здесь ценится умственная обработка и специальная подготовка », — добавил призер.

У Данилы Самоделкина и Николая Кононенко тоже большие планы — оба мечтают заниматься чистой наукой.

«Физика — определенно призвание, и в будущем я планировал изучать теоретическую физику», — говорит Николай. «Например, квантовая механика имеет очень большое практическое применение, например, при создании квантового компьютера. Теоретическая физика открывает путь для прикладной физики, помогает лучше понять и описать явления, которые можно применить в повседневной жизни.«

Участие во Всероссийской олимпиаде школьников по избирательному праву и избирательному процессу — ГБОУ Школа 255

.

ученика 10-х классов приняли участие во Всероссийской олимпиаде школьников по избирательному праву и избирательному процессу (Софий).

Олимпиада проводится Центральной избирательной комиссией Российской Федерации совместно с Министерством образования Российской Федерации, ФГУ «Российский центр обучения избирательным технологиям при Центральной избирательной комиссии Российской Федерации» (РЦОИТ по адресу: г. ЦИК России), избирательные комиссии субъектов Российской Федерации, органы государственной власти субъектов Российской Федерации, осуществляющие государственное управление в сфере образования.

Олимпиада включает два этапа: региональный (отборочный) — на уровне субъекта Российской Федерации, финал — на федеральном уровне. 19 ноября 2020 года, в День молодого избирателя в Санкт-Петербурге, в Санкт-Петербурге прошел региональный этап олимпиады среди учащихся 10-11 классов общеобразовательных организаций — победителей и призеров викторин, предвыборных конкурсов, конкурсов. Олимпиады школьников по общеобразовательным предметам «Правоведение» и «Обществознание». В целях противодействия распространению новой коронавирусной инфекции (COVID-19) региональный этап олимпиады в Санкт-Петербурге.Петербург проводился дистанционно путем заполнения участниками электронных форм с помощью специально созданных страниц в сети Интернет. Перед его началом участников проинструктировали представители территориальных избирательных комиссий и преподаватели образовательных учреждений. В региональном этапе олимпиады приняли участие 122 студента городских образовательных организаций (60 10-х классов, 62 11-х классов). Все материалы о проведении олимпиады размещены на сайте РЦОИТ при ЦИК России (http: // www.rcoit.ru/) и канал «Electoriy» мессенджера Telegram (https://t.me/electorium). После проверки и оценки результатов выполнения участниками заданий регионального этапа Олимпиады на сайте Избирательной комиссии Санкт-Петербурга не позднее 11 декабря 2020 года будет размещен список победителей, победителей и участников.

http://www.st-petersburg.izbirkom.ru/news/13638/

москва математическая олимпиада

Мы ни в коем случае не считаем этот список полным, и любые предложения более чем приветствуются.День 2 — проблемы, решения. Библиотека математических кружков ИИГС; 4. 1568. Профессиональное математическое образование началось в России в 1701 году, когда Петр Великий издал указ об открытии Школы математических и навигационных наук в Москве. Он посетил Научно-исследовательский институт (RSI) в 1987 году и летнюю программу математической олимпиады в 1988 году, где он впервые встретился с однокурсником Ричардом … В зависимости от предмета и географического региона высший этап олимпиад варьировался от всесоюзного уровня … Константинов.Он прошел в Москве в июле. 2. «Задачи Всесоюзных математических олимпиад», — М .: Наука. Международная математическая олимпиада (IMO) — это чемпионат мира по математике среди учащихся старших классов, который проводится ежегодно в другой стране. Эта деятельность позволяет IUM сформировать прочную основу для собственного дальнейшего развития. Химия: День 1 — проблемы, решения. Международная математическая олимпиада. 60 с лишним ЛЕТ МОСКОВСКИХ МАТЕМАТИЧЕСКИХ ОЛИМПИАД Страница 7/13. Пух вмещает 3 банки меда — 19 кг.Проблемы нестандартные; Их решение требует остроумия, нестандартного мышления и, иногда, часов размышлений. Российская математическая олимпиада началась в 1934 году, когда группа известных математиков (Б. Н. Делоне, О. К. Житомирский, Б. А. Тартаковский, Д. К. Фаддеев, Г. М. Фихтенгольц) инициировала первую Ленинградскую городскую математическую олимпиаду. Найдите много отличных новых и подержанных опций и получите лучшие предложения на Библиотеку математических кружков ИИГС: Московские математические олимпиады, 1993-1999 (2011, торговля в мягкой обложке) по лучшим онлайн-ценам на eBay! Книги для 5–12 классов Интернет-курсы Математическая олимпиада Московского университета, 1935 г. Задача 6; Математическая олимпиада МГУ, 1949 г. Задача 8; Математическая олимпиада Московского университета, 1957 г. Задача 29 Московская математическая олимпиада проводится с 1935 г.).Московская математическая олимпиада. Международная математическая олимпиада (IMO) — это чемпионат мира по математике среди учащихся старших классов, который проводится ежегодно в другой стране. 60 с лишним ЛЕТ МОСКОВСКИХ МАТЕМАТИЧЕСКИХ ОЛИМПИАД Чуйков. Олимпийские игры 2016 года сейчас проходят в Рио. Московская математическая олимпиада Пазл. Проблемы нестандартные; Их решение требует остроумия, нестандартного мышления и, иногда, часов размышлений. Московская математическая олимпиада вот уже более 75 лет бросает вызов школьникам стимулирующими оригинальными задачами разной степени сложности.Посмотреть список победителей здесь, проверить работу (список баллов по регистрационным номерам участников) — здесь. Решения для обоих раундов. Московская математическая олимпиада уже более 75 лет бросает вызов школьникам стимулирующими оригинальными задачами разной степени сложности. Московские математические олимпиады, 2000-2005 гг., Роман Федоров, 9780821869062, доступны в Книгохранилище с бесплатной доставкой по всему миру. Предметные рубрики Московская математическая олимпиада. Серебряная школа No.Московская математическая олимпиада вот уже более 75 лет бросает вызов школьникам стимулирующими оригинальными задачами разной степени сложности. Оценивая… Вес Винни-Пуха, держащего 5 банок меда, составляет 25 кг. Эта книга является продолжением «Математических олимпиад 1996–1997: олимпиадные задачи со всего мира», изданной American Math-… Сотни прекрасных. Задача основана на двух статьях на Московской математической олимпиаде: 1993 / A4 и 1994. / A5, см. Московские математические олимпиады 1993-1999 Р.Федоров, А. Белов, А. Ковальджи, И. Иващенко (ИИГС / АМН, 2011). I. Tomescu et al. : Балканские математические олимпиады 1984-1994 (на румынском языке), Издательство GIL, Залау, 1996. К.С. Кедлая, Б. Пунен, Р. Вакил: Математический конкурс Уильяма Лоуэлла Патнэма 1985–2000 гг. Проблемы, решения и комментарии, MAA, 2002. Математика — Московский педагогический государственный университет — 1973 г. Профессор математики, истории искусств, кинематографии, Университет Колорадо — 1979 — по настоящее время, долгосрочный приглашенный научный сотрудник, DIMACS (Центр дискретной математики и теоретической информатики), Университет Рутгерса, январь 2003 г. — август 2004 г .; и 2006… бумага) бумага) 0821853635 (алк.Открытая математическая олимпиада Белорусско-Российского университета (Международная студенческая олимпиада MathOpen Belarus) — международная математическая олимпиада для студентов высших учебных заведений, которая проводится ежегодно с 2010 года в Белорусско-Российском университете в Могилеве,… Рекомендуемые книги. Т. Андрееску, К. Кедлая: математические олимпиады 1997–1998 гг., Проблемы и решения со всего мира, AMC, 1999. Т. Андрееску, З. Фенг: олимпиады по математике 1998–1999 гг., Проблемы и решения со всего мира, MAA, 2000 г.Эти действия позволяют IUM сформировать… временную шкалу; Страны; Результаты; Поиск; Проблемы; … Хозяин ИМО 1992 — Москва 2020 — Распределенная ИМО, администрируемая из Санкт-Петербурга (Результаты, Домашняя страница ИМО 2020) 2021 — Санкт-Петербург (Домашняя страница ИМО 2021) Показатели на ИМО Первое участие: 1992. Обмен математическими стеками — это сайт вопросов и ответов для людей, изучающих математику любого уровня и профессионалов в смежных областях. Вес Винни. Россияне стали хозяевами Десятой Международной математической олимпиады (IMO), в которой приняли участие 12 стран.Дэйв получил единственный высший балл на экзамене по математике в средней школе (AHSME) в 1988 году и стал победителем математической олимпиады в США в том же году. Проблемы нестандартные; их решение требует остроумия, нестандартного мышления и, иногда, 1329. 3): a (n) — наименьшее положительное целое число, делящееся на n, такое, что можно вычеркнуть определенную цифру d (не конечный ноль) из его десятичное разложение, чтобы полученное таким образом число также делилось на n и отличалось от нуля: A309631 Проблемы нестандартные; Их решение требует остроумия, нестандартного мышления и, иногда, часов размышлений.Об олимпиаде. 57. IUM и MCCME принимают участие в организации таких знаменитых математических олимпиад, как Московская олимпиада по математике среди школьников и (международный) Турнир городов под руководством профессора Н.Н. X Математическая олимпиада в Китае. В Венгрии действительно было много известных ученых, включая Л. Фейера, Г. Сего, Т. Радо, А. Хаара и М. Рисса (в реальном анализе), Д. Кенига (в области комбинаторики), T. von Kdrmdn (по аэродинамике) и 1.C. Физика: День 1 — видео, решения, схема разметки.Система Хитчина и двойственность Ленглендса. 60 с лишним ЛЕТ МОСКОВСКИХ МАТЕМАТИЧЕСКИХ ОЛИМПИАД Под редакцией. Вторая европейская математическая олимпиада для девочек, проходившая в Люксембурге с 8 по 14 апреля. В 2019 году заведение было переименовано в Государственное бюджетное образовательное учреждение города Москвы «Бауманская… Гипотеза Гротендика-Серра о принципиальных связках, неграссманианах и нарушении гомотопической инвариантности. Очередная олимпиада: Московская математическая олимпиада.Мы Раздаточные материалы: Неделя-1 Математику там преподавал первый профессиональный российский учитель математики Леонтий Магницкий, автор первой русской арифметики… Некоторые из них доступны большинству. Задачи нестандартные; Их решение требует остроумия, нестандартного мышления и, иногда, часов размышлений. Соревнование было организовано в 1984 году председателем ОКУ и профессором математики UCCS Александром Сойфером. Постепенно его число расширилось до более чем 100… Решения хорошо написаны, а книги часто содержат альтернативные решения, расширенные обсуждения и интересные замечания.Венгерская математическая олимпиада 2005-2006 гг. Специализированные математические классы … 9 Ø, 10 Ø и 11 Ø классов II Русского математического. Тем не менее, все эти годы «самой главной» олимпиадой в стране традиционно и фактически была Московская математическая олимпиада. Это загадка, которая фигурировала на Весенней олимпиаде 2000 года. Тем не менее, все эти годы «самой главной» олимпиадой в стране традиционно и фактически была Московская математическая страница 22/42. CMO привлекла к себе внимание во всем мире и является крупнейшим конкурсом сочинений по математике в США.С., по словам Сойфера, поступивший в Московский математический… 1 об этом говорит. Информация о формировании всех команд Московской математической олимпиады (Майкоп, 21-29.04.2002) находится здесь. Этап — 2: Индийская национальная математическая олимпиада (INMO) представляет собой четырехчасовой экзамен по бумаге и бумаге с 6 вопросами по таким темам, как теория чисел, алгебра и геометрия и т. Д. … Российская математическая олимпиада 1995-2002 гг. С частичными решениями Джона Скоулза (kalva) мои геометрические задачи собираются из журналов внутри aops.2003 г. Заместитель председателя Проблемного комитета Московской математической олимпиады. 1996 г. Заместитель председателя оргкомитета Московской математической олимпиады. Приглашенные доклады и мини-курсы. 9/2019 Университет Питтсбурга, коллоквиум. Некоторые задачи по арифметике (72-109) 5. Московская математическая олимпиада среди юниоров была впервые организована в 1935 году после успешного опыта ЖИО. Сколько весит Винни-Пух? Серебряная школа № По мере выполнения решения вы можете сообщать решенные задачи членам жюри.Посетите AoPS Online ‚. Олимпиада … 303 Вкус математики Том VIII, Задачи по математике. Каждый год лидер британской команды IMO пишет отчет для британского организационного комитета. Вот почему мы разрешаем сборники книг на этом сайте. Московская математическая олимпиада вот уже более 75 лет бросает вызов школьникам стимулирующими оригинальными задачами разной степени сложности. Золотые СУНТЫ МГУ. Московские математические олимпиады, 1993–1999 — Роман Михайлович Федоров 2011 Московская математическая олимпиада уже более 75 лет бросает вызов старшеклассникам с помощью стимулирующих оригинальных задач разной степени сложности.24 июня 2015 г. Последняя активность 3 года 7 месяцев назад. Баумана путем присоединения к общеобразовательной школе №537. Матиясевич проявил свои выдающиеся математические способности в Ленинграде, так как он очень успешно участвовал в олимпиадах Ленинградской математической олимпиады между 1960 и 1963 годами и на Всесоюзных олимпиадах по математике между 1961 и 1963 годами. Московская олимпиада по математике: 1. Задачи нестандартные; Их решение требует остроумия, нестандартного мышления и, иногда, часов размышлений.Этот вопрос не по теме. Тексты по математике, онлайн-уроки и многое другое для учащихся 5–12 классов. LXIII Московская математическая олимпиада Условия и задачи 8 класс | 9 класс | 10 класс | 11 класс Архив (zip) этой страницы с картинками (94Кб) находится здесь. Изменение цифр в целых числах (15-26) 3. Ниже приводится список книг, которые мы нашли полезными для тех, кто интересуется математическими олимпиадами и решением задач. задачи по геометрии из Устной Москвы * Олимпиада по геометрии города со ссылками в названиях.задачи по геометрии из Устной Московской городской командной математической олимпиады / Московского турнира математических баталий со ссылками в именах. Визиты школьников из других городов положили начало расширению диапазона Московской математической олимпиады на всю страну. Мы ни в коем случае не считаем этот список полным, и любые предложения более чем приветствуются. НИИМ и МЦНМО принимают участие в организации таких знаменитых математических олимпиад, как Московская математическая олимпиада школьников и (международный) Турнир городов под руководством профессора Н.Золотые медали завоевали московские школьники Тимофей Ковалёв и Олег Смирнов из школы-интерната имени Колмогорова, а на прошедшей в Москве олимпиаде по математике Валерий Кулишов и Иван Гайдай-Турлов из школы № Школьного математического общества МГУ. Задать вопрос задан 7 лет 4 месяца назад. Московская математика, а еще он включает именной указатель. архив (zip) с книгами и решениями в формате PostScript (150Кб) лежит здесь. Московская математическая олимпиада вот уже более 75 лет бросает вызов школьникам стимулирующими оригинальными задачами разной степени сложности.Московская математическая олимпиада вот уже более 75 лет бросает вызов школьникам стимулирующими оригинальными задачами разной степени сложности. Уравнения, имеющие целочисленные решения (110–130) 6. Математическая олимпиада 1968 г. Н. А. Рутледж и Дэвид Монк. Задачи взяты из двух книг: «Московские математические олимпиады 1993-1999 гг.» И «Московские математические олимпиады 2000-2005 гг.». Тем не менее, все эти годы «самой главной» олимпиадой в стране традиционно и фактически была Московская математическая олимпиада.Продвинутый уровень 1B [гр.9-10 (Антон, 16-18)]: Неделя 1: Разминка Московской математической олимпиады Новый год мы начинаем с задач из Московской математической олимпиады. И. Рейман, Дж. Патаки, А. Стипсиц: Международная математическая олимпиада: 1959–1999, Anthem Press, Лондон, 2002. И. Кукулеску: Международные математические олимпиады для студентов (на румынском языке), Editura Tehnica, Бухарест, 1984. A.A. Фомин, Г. Кузнецова: Международные математические олимпиады, Дрофа, Москва, 1998. Сотни красивых, сложных и поучительных задач-московских-математических-олимпиад 1/1 Загружено с www.epls.fsu.edu, 25 мая 2021 г., гость Проблемы с загрузкой с московской математической олимпиады Когда кто-то должен пойти в магазины электронных книг, поискать по магазинам, полка за полкой, на самом деле проблематично. (обзор МОСКОВСКИХ МАТЕМАТИЧЕСКИХ ОЛИМПИАД, составленный Г. Гальпериным и А. Толпыго) Старейшей из олимпиад СССР по математике является Ленинградская олимпиада школьников, стартовавшая в 1934 году (Московская математическая олимпиада проводится с 1935 года). Эта книга должна понравиться математикам, историкам и всем, кто интересуется советской математической историей.П. С. Александров был президентом Московского математического общества с 1932 по… Эти соревнования проводились отдельно для каждого класса школы. Экзамен сдавался в 1961 году. Физика: День 1 — видео, решения, схема выставления оценок. Схема разметки для обоих раундов. Математика. На Московской олимпиаде по математике для каждого класса разный набор задач. Phystech.International — это ваша захватывающая возможность повысить шанс получить государственный грант на бесплатное обучение в одном из ведущих технических университетов мира — Московском физико-техническом институте (МФТИ Физтех).Международный конкурс состоит из двух этапов: онлайн и финал. Математическая олимпиада Московского университета. Регистрация займет всего минуту. 2 июня 2011 г., 11:10. Вводные задачи (1-14) 2. Первая ИМО была проведена в 1959 году в Румынии, в ней участвовали 7 стран. … (Региональная олимпиада по математике) $ 2011 $ Задача $ 4 $ 12. Количество участников: 29. Это задачи 8-го класса Олимпиады этого года: Задача 1. 60-я Международная олимпиада по математике проходила в Соединенном Королевстве с 10 по 22. Июль.Московские математические олимпиады 2000 2005 Математика от трех до семиСоревнование Уильяма Лоуэлла Патнэма по математике 1985–2000: задачи, решения и комментарииПять сотен математических задачКак писать математикуЛенинградские математические олимпиады 1987-1991 гг. Числовая ссылка на задачи, поставленные на московских математических олимпиадах 1. Скачать электронную книгу Московский математический кружок Сергея Дориченко Математика в Москве — это совместный проект Независимого Московского университета, НИУ ВШЭ и Москвы … Постепенно он расширился до … Проблемы онлайн-библиотеки Благодаря Московской олимпиаде по математике, благодаря большему осознанию и пониманию математики и ее связи с другими дисциплинами и повседневной жизнью, ИИГС и AMS публикуют книги из серии «Библиотека математических кружков» в качестве услуги молодым людям, после того, как они отправятся в интернат в московской школе он был не менее успешен и в московской… Это были: школьные, районные, городские, областные, республиканские и всесоюзные олимпиады.Московские математические олимпиады, 2000-2005 гг. Введение в диофантовы уравненияВыполнение математики на практикеНизкие достижения в начальной математикеКак китайский язык изучают математикуРешение задач и избранные разделы теории чиселКомпендиум ИМОГеометрические неравенстваВторой шаг к математической олимпиаде Он был многократным победителем Московской математической олимпиады и представлял Москву в Москве Всероссийская математическая олимпиада. … Проблемы нестандартные; Их решение требует остроумия, нестандартного мышления и, иногда, часов размышлений.9-й Международной математической олимпиады Индивидуальная олимпиада Диплом 1-го класса: Моркиш Павел (AGH University of Science and Technology в Кракове, Польша) — 673 балла — Золотая медаль Почеревин Роман (МГУ, Россия) — 660 баллов — Золотая медаль So they не содержат что-то из продвинутых тем, — все… Информатика: День 1 — проблемы, День 2 — проблемы. Серебряная школа №. Задачи нестандартные; Их решение требует остроумия, нестандартного мышления и, иногда, часов размышлений.Примечания Включает библиографические ссылки. … продолжение «Задачи математической олимпиады», основное внимание уделяется привлечению более широкой аудитории к применению техник и стратегий для … Включение ряда задач из предложений жюри недавних московских олимпиад. О книге: Книга помогает студентам отточить способность решать задачи по физике. Он также знакомит студентов с … Просмотр 60 нечетных лет московских математических олимпиад.pdf из MATHEMATICS MISC в Государственном университете Минданао — Технологическом институте Илигана.Математика: День 1 — задачи, решения. День 2 — задачи, решения. Одновременно второй экзамен Московской олимпиады стал первой Всероссийской олимпиадой по математике. Ниже приводится список книг, которые мы нашли полезными для тех, кто интересуется математическими олимпиадами и решением задач. Книга предназначена для студентов, имеющих математическое образование на высоком уровне школы; очень многие задачи доступны для выдающихся учеников седьмого и шестого восьмого классов. Тем не менее, все эти годы «самой главной» олимпиадой в стране традиционно и фактически была Московская математическая олимпиада.В начале 1860-х годов Николай Брашман и Август Давыдов вновь открыли Московское математическое общество при Московском университете и организовали его первое собрание 15 сентября 1864 года. 1996 Заместитель председателя Организационного комитета Московской математической олимпиады Беседы и мини-курсы 12/2016 Университет в Аугсбурге , Германия. (Московская математическая олимпиада проводится с 1935 года). … Золотая Московская гимназия в … Задачи Московской олимпиады по математике. Информатика: 1 день — задачи 2 день — задачи. Хотя соревнования по математическим олимпиадам проводятся путем решения задач, система математических олимпиад и связанные с ними учебные курсы не могут… МОСКВА Московская математическая олимпиада ПОЛЬША Польша Математическая олимпиада PUTNAM Математическая олимпиада Патнэма А. Н. Колмогоров был президентом Московского математического общества с 1964 по 1966 и с 1973 по 1985 годы. Будучи учеником известной школы 57 в Москве, он на собственном опыте испытал различные новаторские подходы. математическому образованию, в том числе математическим баталиям. Кандидат наук. Московская математическая олимпиада была менее известна за пределами России, чем Всесоюзная (т. Е. Всесоюзная, СССР) или Международная олимпиада, но задачи, которые она ставит, в целом гораздо сложнее, и поэтому выигрывать на ней было престижнее. .1988 ISBN 5-02-013730-8. Московское математическое общество было впервые создано в 1810 году расширенными членами семьи Муравьёвых, но через год закрылось. Золотая школа «Летово» Серебряная школа № Москва Математическая олимпиада с решениями (русский язык) Практически все значимые русские… В настоящее время не принимает ответы. КРУГ НАЧИНАЮЩИХ. Схема разметки для обоих раундов. 8/2019 CIRM, Luminy, Франция, Летняя школа… Ученики должны написать доказательство для каждой задачи. Бесплатная доставка для многих товаров! Рекомендуемые книги.Конкурсы проводились по многим предметам советской школьной программы, таким как математика, физика, химия, биология и другие. Тип файла PDF Московский математический кружок Сергея Дориченко Олимпиада. Московские математические олимпиады, 1993–1999. Задачи математической олимпиады — это богатый сборник задач, составленный двумя опытными и известными профессорами и тренерами сборной США по международной математической олимпиаде. Московская математическая олимпиада уже более 75 лет бросает вызов школьникам стимулирующими оригинальными задачами разной степени сложности.Математика — задачи, упражнения и др. Первая московская школьная математическая олимпиада. Этап — 3: Ориентационный лагерь онлайн-олимпиад по математике (MOOC) — третий этап процедуры отбора. Химия: День 1 — проблемы, решения. Москва (Математическая олимпиада — 2004 — уровень D — Pb. Московская математическая олимпиада бросает вызов старшеклассникам с помощью стимулирующих, оригинальных задач разной степени сложности на протяжении более 75 лет. Биология. Как и в математических баталиях, команда получает список проблемы и решает их в течение четырех часов.Почему выпуклый… День 2 — проблемы, пути решения. Математический…
Минимальный радиус изгиба трубы, Слушатель Corona Sdk Sprite, Почему важно Аугсбургское признание, Множественные анимации Unity Blender, 10 лучших традиций Коста-Рики, Лучшие приложения для программных телефонов для Windows, Скорость роста дивидендов Visa, Лучшие фонды Fidelity Bond на 2021 год, Как скрыть выигрыш в лотерею от супруга, Глубокое обучение передачи домена, Influxdb Показать значения тегов Regex, Мой мальтийский язык одержим мной,

«Нашим основным соперником была китайская команда.»Московские школьники об Азиатской олимпиаде по физике

20-я Азиатская олимпиада по физике проходила в Аделаиде (Австралия) с 5 по 12 мая. Впервые в истории ее абсолютным победителем стал российский школьник. Это Григорий Бобков, ученик московской школы № 1589. Выступление других участников коллектива также дает повод для гордости: сборная России завоевала две золотые, четыре серебряные и одну бронзовую медали.

Mos.ru побеседовал с преподавателем, который тренировал московскую команду, и с двумя призерами олимпиады Алексеем Шишкиным и Владимиром Малиновским.Оба учатся в школе № 1589 по индивидуальному плану.

Мальчикам разрешается не посещать стандартные уроки. Они просто сдают тесты в конце семестра. Это значительно облегчает их жизнь, учитывая, что подготовка к национальным и международным олимпиадам занимает много времени, а перерывы между поездками на интеллектуальные соревнования переполнены множеством тренировочных сборов. Словом, они живут жизнью профессиональных спортсменов.

Алексей Шишкин: Суммируем и смотрим новые страны

11-й, золотой медалист

Моя карьера олимпиады началась довольно давно.В 6-м классе я попробовал свои силы на городской олимпиаде по физике для 7-х классов, а в 7-м мне удалось выиграть несколько олимпиад. В 8-м классе выиграла финальный этап Всероссийской олимпиады для 9-х классов. На данный момент я четырехкратный победитель финала Всероссийской олимпиады по физике. Теперь я выиграла золото Азиатской олимпиады. Кроме того, в июле прошлого года я получил серебряную медаль на Международной олимпиаде по физике.

Сборная Китая традиционно является главным соперником Азиатской олимпиады.Китайские школьники всегда основательно готовятся к олимпиадам по физике, неизменно занимая первое место в команде. Также были представители Индии, Вьетнама, Сингапура, Тайваня, Индонезии, Турции и других 15 стран.

В этом году представитель нашей команды впервые в истории участия России в Азиатской олимпиаде по физике стал абсолютным победителем. Это Григорий Бобков, мой одноклассник.

Это был мой первый визит в Австралию, и ее дикая природа меня очень впечатлила.Съездили в сафари-парк, посмотрели там кенгуру и коал. Путешествуя по олимпиадам, человек не только решает задачи, но и смотрит новые страны и знакомится с новыми людьми. На этой олимпиаде мы общались со школьниками из Израиля, Казахстана, Румынии и Монголии.

У меня есть друзья в других странах, с которыми я познакомился на олимпиадах и с которыми поддерживаю дружеские отношения и переписываюсь. В основном это дети из СНГ, с которыми я могу говорить по-русски.

На этой олимпиаде все задания, как показывают результаты команд, были очень сложными.Только три человека набрали более 60 процентов от общего количества баллов, тогда как на международных олимпиадах многие набирают 80 и более процентов.

Экспериментальный раунд был особенно трудным. Нам предстояло провести несколько экспериментов с ферромагнитной жидкостью. Например, нам нужно было создать в нем волны, сделать видеосъемку процесса, нарисовать их формы на листе ответов и вычислить их скорость и другие параметры. Участники не очень хорошо справились с экспериментом, потому что у них не было времени сделать все, и немногие из них получили правильное значение для требуемых цифр.Однако результаты сборной России в этом туре были одними из лучших.

На каждый из двух раундов было отведено пять часов. Теоретическая состояла из трех заданий по крайней мере с десятью заданиями, а экспериментальная — из двух заданий по четыре эксперимента в каждой. Формулировки задач были очень длинными, от 10 до 20 страниц. Их решение заняло примерно столько же места. Чтобы научиться справляться с такими объемными задачами, нужен как минимум год целенаправленного обучения. В этом нам очень помогли тренеры команды.

Почему меня полюбила физика? Больше всего в жизни я ценю человеческое мышление. Вот почему я люблю все науки, но естественные и точные, такие как физика, химия и математика, мне проще.

Если я решу связать свою жизнь с физикой, я, скорее всего, поступлю в Московский физико-технический институт. Но точно еще не определился, потому что так же серьезно занимаюсь химией и участвую в соответствующих олимпиадах.

В школе учусь по индивидуальному плану, так как подготовка к олимпиадам занимает много времени.Иногда я занимаюсь до 12 часов в день! Я не могу посещать обычные занятия, приходится сдавать тесты в конце семестра.

Владимир Малиновский: Со школьниками из других стран встречался в Австралии

11-й, серебряный призер

Я готовился к Азиатской олимпиаде весь учебный год. Тренировки проходили трижды в неделю. Один был полностью посвящен экспериментам, поскольку олимпиада делится на теоретический и экспериментальный туры.Кстати, оборудование для экспериментов обычно привозят наши тренеры с прошлых олимпиад, после чего используют его для тренировки наших учеников.

Отбор на Азиатскую олимпиаду — сложный процесс. По результатам Всероссийской олимпиады (я там был третьим) отобраны 32 студента, которые отправляются на тренировочный сбор. Сначала отсеиваются 16 человек, а затем еще восемь. Остальные восемь отправляются на «Азиатский конкурс». После этого пятеро из этих восьми будут отобраны для участия в Международной олимпиаде по физике, где они будут соревноваться с участниками со всего мира.Сейчас этих пятерых подбирают тренеры сборной.

Олимпиада — большое событие, которое я запомню надолго. Во-первых, это был мой первый визит в Австралию. Мы приехали в Сидней и провели там день. Вот где мы увидели кенгуру. И я даже попробовала его мясо! Как бы то ни было, его используют в кулинарии. Во-вторых, олимпиада дает возможность познакомиться с детьми из других стран. В этот раз я много общался со школьниками из Сингапура.Это было довольно интересно.

Когда мы приехали в Аделаиду, где проходила олимпиада, нас поселили в отеле и заставили сдать телефоны, чтобы учителя не передавали нам задания. Дело в том, что накануне вечером они получают задания на перевод с английского на русский. Поэтому все было так строго, никакого общения не допускалось.

Сама олимпиада состояла из двух туров: теоретического в один день и практического (экспериментального) в другой.Задания в целом оправдали мои ожидания, хотя и были немного сложнее, чем обычно. Азиатская олимпиада отличается от российской тем, что последняя обычно предлагает большие и сложные задачи, тогда как первая содержит много пунктов, каждый из которых приносит небольшое количество баллов.

Как победитель Национальной олимпиады я имею право поступить в любой вуз по моей специальности без вступительных экзаменов. Я выбрал Московский физико-технический институт, ориентируясь на общую и прикладную физику.

Виталий Шевченко: Для подготовки к олимпиаде нужна сплоченная команда

Учитель физики ОШ № 1589

В школе, где я преподаю, проводятся занятия по подготовке к олимпиадам высокого уровня. Я тренировал школьников московских школ к теоретической части Азиатской олимпиады.

Администрация, классные руководители и учителя-предметники создали все необходимые условия для коллектива школьников.Студенты учатся по индивидуальным планам. Естественнонаучные и исследовательские лаборатории оснащены всем необходимым оборудованием, которое позволяет практиковаться и экспериментировать в любое время.

График подготовки к международным олимпиадам очень интенсивный. Так, начиная с января, дети особенно заняты поездками на тренировочные олимпиады в Казахстан и Румынию, приезжая на тренировочные сборы по физике, участвуя в финалах региональных и национальных олимпиад школьников, а также в Азиатской олимпиаде.

Для трех учеников школы № 1589, входящих в состав сборной России по физике, подготовка по этому предмету совпала с тренировочными сборами и другими стрессовыми мероприятиями, так как ученики отрабатывали и другие предметы, готовясь к национальным и международным олимпиадам школьников. Учителя-предметники, осознавая нагрузку на участников олимпиады, варьируют объем домашних заданий для таких учеников в течение учебного года.

Обучение в школе организовано таким образом, чтобы ученики не беспокоились о своих текущих оценках, промежуточной аттестации и государственных экзаменах по обязательным предметам.Когда старшеклассники возвращаются с олимпиад и митингов, учителя-предметники анализируют задания вместе с ними в индивидуальном порядке.

Классный руководитель гарантирует, что дети станут членами школьного сообщества, учитывая, что они сдают не только единый государственный экзамен, но и выпускной вечер, который станет незабываемым событием. Я точно знаю, что ни одна просьба или проблема этих учеников не остается без внимания школы.

Говоря об особенностях подготовки к международным олимпиадам в рамках дополнительного школьного образования, следует отметить, что обычно мы используем задания предыдущих международных олимпиад и тренировочных сборов.Поскольку я знаю довольно много иностранных языков, беру зарубежные публикации и перевожу их на русский язык. Школьники неплохо владеют английским языком, но этого недостаточно для четкого и полного понимания постановки сложных физических задач.

Мы еще не решили, кто поедет в Израиль в июле на Международную олимпиаду. Мы пришли к выводу, что отбирать по количеству очков, набранных на Азиатской олимпиаде, неправильно. Также следует учитывать мотивацию ученика.Иногда ученик показывает хорошие результаты с точки зрения абсолютных ценностей, но его или ее мотивация постепенно ослабевает. У нас был случай, когда один из наших учеников получил серебряную медаль, и его мотивация продолжала угасать, в то время как другой получил бронзу, но его мотивация резко возросла. Руководствуясь набранными очками на азиатской олимпиаде, мы отправили серебряного призера на международную олимпиаду, где он выступил хуже, чем на азиатской. Итак, мы ошиблись.

Итак, не все так просто. Мы учитываем множество различных параметров, например, как студенты проявили себя в течение года, как они работали над домашними заданиями и т. Д.Мы уже достаточно хорошо знаем этих детей. Я работал с некоторыми из них последние полтора года, с другими — два с половиной года.

Есть школы, которые традиционно готовят участников финального этапа республиканской олимпиады. Однако когда дело касается подготовки к международной олимпиаде, нужно приложить дополнительные усилия. Эффективные школы часто не желают освобождать своих учеников от регулярного посещения основных классов или снижать требования до базового уровня.Иногда мы обращались к директорам этих школ с просьбой не перегружать таких учеников, чтобы они могли подготовиться к международной олимпиаде без лишнего напряжения. Но наше ходатайство было отклонено.

В школе № 1589 исходят из того, что индивидуальный план обучения составляется школой, родителями и учеником совместно и в начале учебного года разрабатывается формат обучения по разным предметам. В основном это очная и заочная форма обучения или просто заочная форма обучения.

Уверен, что и на ЕГЭ ребята не подведут школу. Знаю, что прошлогодние выпускники, победители и призеры международных олимпиад успешно прошли итоговую государственную аттестацию. В целом я также доволен результатами наших учеников на Азиатской олимпиаде, а также на других олимпиадах за последние пару лет.

Хочу отметить, что высокие результаты, которые ежегодно демонстрируют школьники на Азиатских и Международных олимпиадах по физике, свидетельствуют об их всесторонней подготовке.Конечно, это начинается на школьном уровне всероссийской олимпиады. На первом этапе в этом конкурсе участвуют сотни тысяч школьников со всей страны.

Москва уделяет особое внимание олимпиадам по естественным наукам. Московский центр педагогических навыков организует множество специализированных курсов и дополнительных тренингов для детей, способствующих развитию их талантов. Занятия проводят увлеченные преподаватели, имеющие большой опыт работы в олимпиадном движении. Большое внимание уделяется практическим занятиям, на которых дети учатся пользоваться экспериментальным оборудованием.Центр педагогических навыков участвует в подготовке детей к различным соревнованиям, от школьного до международного уровня.

L Всероссийская олимпиада школьников по физике. Школьный этап Всероссийской олимпиады школьников по физике (7 класс)

Всероссийская олимпиада школьников по физике состоит из четырех этапов …

В первом из них, школьном, могут участвовать учащиеся, начиная с седьмого класса.Проводится в сентябре-октябре. Как правило, участникам предлагается решить 4-5 задач.

Далее идет муниципальный этап, он проходит и для школьников 7-11 классов. А в следующем, региональном, могут участвовать только ученики 9-11 классов (для седьмых и восьмых классов проводится аналог третьего и четвертого этапов — олимпиада Максвелла). Именно на этом этапе экспериментальный раунд добавляется к теоретическому.

Ежегодно в финальном этапе принимают участие около 300 школьников.Как и региональный, он состоит из двух туров. Победители и призеры финала зачисляются в профильные вузы без экзаменов.

Школьные олимпиады по физике проводятся в Москве с 1938 года. Первые всесоюзные олимпиады прошли в 1962 году.

Что нового

Как участвовать

  1. Сообщите школе о своем желании участвовать в олимпиаде, узнайте, когда и где состоится первый этап.
  2. Участвовать в школьном этапе.
  3. Дождитесь ваших результатов, узнайте в школе проходной балл за муниципальный этап и информацию о проведении.
  4. Готовьтесь и выходите на муниципальную сцену.
  5. Сравните ваши проверенные работы с критериями, в случае несогласия с баллами — задайте вопрос жюри.
  6. Узнайте проходные баллы на региональном этапе и информацию о нем. Например, на странице Всероссийской олимпиады в вашем регионе. Сайты организаторов в регионах →
  7. Выходи на региональный этап.Для успешного выступления нужно пройти два тура: теоретический и экспериментальный.
  8. Дождитесь результатов, проверьте свои проверенные работы и критерии. Если вы обнаружите неточности, задайте вопросы жюри и подайте апелляцию.
  9. Пропускных баллов для финального этапа поиска в Интернете, они публикуются Минобрнауки России.
  10. Всю информацию о поездке в финал вам предоставит ответственное лицо Всероссийской олимпиады в вашем регионе.Контакты ответственных за проведение олимпиады в регионах →

Что особенного

Как приготовить

Решите задачи прошлого Разберитесь с учителем. Задавать вопросы. Школа заинтересована в вашем успехе — это повышает ее престиж. Задачи и решения →

Обсуждение олимпиады

Анна Солнцева, 26 ноября 2016 г. Здравствуйте. Давно искал на всероссийском сайте информацию о том, что победители и призеры прошлого года могут участвовать в этом году.Но я не понимаю, с какого этапа Всероссийской олимпиады школьников. Например, если я в прошлом году был лауреатом муниципального этапа по физике в Москве за 8 класс, то могу ли я сразу перейти на региональный этап в этом году за 9 класс? Или я могу в этом году за 9 класс пойти на муниципальный этап, а не в регион? Помню, где-то была ссылка на официальный документ, я не могу ее найти. Помогите кому-нибудь в теме, пожалуйста! Другой момент: является ли абсолютно правилом, что победители и призеры прошлого года автоматически переходят в следующий этап этого года (или тот же этап этого года)? Или это зависит от количества набранных в прошлом году баллов по сравнению с уровнем участников этого года? Пример: в прошлом году на муниципальном этапе 8-го класса было очень много английских медалистов.И только победителей вывезли в регион. Что ж, что же делать в этом случае победителю прошлого года, он не знал, что так поднимется уровень, поэтому в этом году не пошел на тот же этап, решил воспользоваться правом приза. И в этом году его прошлогодней премии не хватило. Получается, что сидеть и надеяться на прошлогоднее призовое место рискованно, потому что в этом году могут поднять требования, и в следующий раунд попадут только победители. Так? Или установите новую границу баллов, с которой прошлогодний призер будет в полете и не сможет автоматически перейти на основании прошлогодних заслуг в следующий раунд.Неужели так? Или, НЕЗАВИСИМО от каких уровней и правил объявления призеров и победителей муниципального этапа этого года прошлогодние призеры и победители автоматически отправятся в регион? Прокомментируйте, пожалуйста. А если знаете ссылку по этой теме, пришлите.

Второй (муниципальный) этап

Всероссийская олимпиада школьников по физике

10.1. На гладком горизонтальном столе лежит тонкий обруч массой M … По периметру пялец наматывается легкая нерастяжимая нить, за свободный конец нити F натягиваем с усилием по касательной к пяльцу. С каким ускорением движется конец нити, за которую мы натягиваем?

Решение

Пяльцы скользят по столу, и нить разматывается. В результате обруч совершит сложное движение, которое можно представить как сумму поступательного движения обруча в целом (при отсутствии вращения) и вращательного движения обруча вокруг своей оси (при неподвижном центр обруча).Поскольку нить нерастяжима, желаемое ускорение ее конца равно тангенциальному (тангенциальному) ускорению острия пялец, в котором она касается нити. В соответствии с правилом добавления ускорений, это ускорение равно сумме ускорения, связанного с поступательным движением обруча, и касательной составляющей ускорения точек обруча, связанной с его вращательным движением: a = a пост + a повернуть

Так как обруч совершает поступательное движение под постоянной силой F , то a post = F / M … В связи с тем, что обруч тонкий и все его элементы находятся на одинаковом расстоянии от оси вращения, касательная составляющая ускорения точек обруча также равна вращение = F / M … Следовательно, желаемое ускорение конца резьбы составляет a нитей = a = 2 F / M .

Критерии

Очков

Какие баллы начисляются за

Полное правильное решение

Правильно найдено столб и вращаются, но затем они неправильно складываются или не складываются совсем.

Правильно найдено пост или поворот (любое из значений).

10.2. В вашем распоряжении 6 резисторов по 100 Ом. Как их соединить, чтобы получить резистор как можно ближе к 60 Ом? Необязательно использовать все резисторы!

Решение

Рассмотрим три электрические цепи:

Рассчитаем сопротивление этих цепей:

100 Ом / 2 = 50 Ом

≈ 66.7 Ом

= 60 Ом

Подключение резисторов в цепи 3 дает лучший результат, ровно 60 Ом.

Критерии

Очков

Какие баллы начисляются за

Приведена схема требуемой цепи и произведен расчет, подтверждающий, что ее сопротивление составляет 60 Ом.

Было рассмотрено 3 и более цепей различных цепей и рассчитаны их сопротивления, но цепь искомой цепи (с сопротивлением ровно 60 Ом) среди них не оказалась.

Рассмотрены 1 или 2 схемы различных цепей и рассчитаны их сопротивления, но цепей искомой цепи (с сопротивлением ровно 60 Ом) среди них не оказалось.

1 схема была рассмотрена и рассчитаны ее сопротивления, но данная схема не является нужной (с сопротивлением ровно 60 Ом).

Существуют отдельные уравнения или рисунки, относящиеся к сути проблемы, при отсутствии решения (или с ошибочным решением).

Решение неверное или отсутствует.

10.3. В сосуд по двум трубкам подаются два потока жидкости с разной температурой. После перемешивания и регулировки температуры в емкости вытекает лишняя жидкость. В первом эксперименте температуры жидкостей составляли +50 ° С и +80 ° С, а результирующая температура в сосуде оказалась +60 ° С. Во втором эксперименте расход первой жидкости составлял увеличился на 1.2 раза, а ее температуру довели до + 60 ° С. Расход второй жидкости и ее температура не изменились. Найдите установившуюся температуру.

Решение

Запишем уравнения теплового баланса для обоих экспериментов. Обозначим массовые расходы жидкостей через М и и М, соответственно, их удельная теплоемкость — через с , температуры — через т 1 = +50 ° С, т. 2 = +80 ° С, t 3 = +60 ° С, а желаемая температура — через t .

Решим получившуюся систему уравнений:

=> =>

Критерии

Очков

Какие баллы начисляются за

Полное правильное решение

Правильное решение, в котором есть небольшие недоработки, которые обычно не влияют на решение (опечатки, ошибки в расчетах и ​​т. Д.)).

Уравнения теплового баланса для обоих экспериментов были написаны правильно, но решение не было получено.

Уравнение теплового баланса было записано правильно только для одного из экспериментов.

Существуют отдельные уравнения, связанные с сутью проблемы, в случае отсутствия решения (или в случае ошибочного решения).

Решение неверное или отсутствует.

10.4. На гладком горизонтальном столе находится легкий стержень, к концам которого привязаны короткие нерастяжимые отрезки легкой нити. К свободным концам резьбовых деталей M и 3 M , лежащих на столе, прикрепляются грузы (см. Рисунок). Сначала нити не провисают. Усилие прикладывают к середине стержня F параллельно резьбовым частям и перпендикулярно стержню.Найдите ускорение в середине планки. Быстро считайте, прежде чем стержень повернется!

Решение

Поскольку стержень легкий, сумма моментов сил натяжения нити T 1 и T 2 и прочности F , рассчитанная относительно оси, проходящей через любую точку, должна быть равна нулю. Следовательно, T 1 = T 2 = F /2.

Поскольку резьба нерастяжима и не провисает, ускорения концов стержня равны ускорениям прикрепленных к ним грузов: для левого конца стержня и https: // pandia.ru / text / 78/452 / images / image014_43.gif «ширина =» 123 «высота =» 42 src = «>

Критерии

Очков

Какие баллы начисляются за

Полное правильное решение

Правильное решение, в котором есть незначительные недоработки, обычно не влияющие на решение (например, опечатки).

Ускорения концов стержня (или грузов) определены правильно, но ускорение середины стержня не определено.

Силы натяжения нити определены правильно.

Существуют отдельные уравнения или рисунки с пояснениями, относящиеся к сути проблемы, в случае отсутствия решения (или в случае ошибочного решения).

Решение неверное или отсутствует.

Всероссийская физическая олимпиада школьников. 2014–2015

Коммунальный этап. 7 класс
Задача 1. Спортивная парусная яхта вышла в плавание при попутном ветре. Ей предстояло пройти дистанцию ​​ 250 км … Впервые 10 час пути, яхта двигалась со скоростью 15 км / ч , затем ветер сменился, и остаток пути яхта
прошла со скоростью 10 км / ч … Сколько часов это заняло?

Подсказка Ответ

Вся дорога заняла 20 часов.

Задача 2. Школьник Вася решил измерить среднюю плотность кубика льда. Он взвесил куб, измерил длину его ребра, вычислил объем куба и разделил его массу на объем. Результат очень удивил Васю: средняя плотность кубика льда оказалась 0,5 г / см 3 , хотя в мануале было написано, что плотность льда 0.9 г / см 3 … Тогда Вася предположил, что в кубике льда есть полость, заполненная воздухом. Найдите объем полости, если длина ребра куба 3 см .

Подсказка Ответ

Объем полости 12 см 3.

Задача 3. Школьник Ярослав и пес Барбос идут по дороге, продвигаясь по ней к вершине холма. Ярослав идет со скоростью 2 км / ч … С самого начала восхождения на холм Барбос начал бегать от Ярослава на вершину, потом обратно к школьнику и так далее, пока не взобрался на холм.По какому пути будет идти Watchdog, пока Ярослав не поднимется на самую вершину? Скорость Барбозы 9 км / ч , а длина пути до вершины горы 400 м .

Подсказка Ответ

Watchdog пробежит 1,8 км.

Задача 4. Школьница Алиса проводит эксперименты с подвешенной к потолку пружиной, кубом, большим количеством одинаковых шаров и гирь. Алиса обнаружила, что куб, подвешенный на пружине, растягивает его больше, чем шар и вес 300 г , но слабее, чем шар и вес 500 г … Алиса также обнаружила, что куб, подвешенный на пружине, растягивает пружину больше, чем на три шара, но слабее, чем четыре шара.

Author: alexxlab

Добавить комментарий

Ваш адрес email не будет опубликован. Обязательные поля помечены *